prove existence of the limit of a sequence











up vote
3
down vote

favorite












So I have the following problem:
$ x_0 = 1 , x_1 = 2 , $and $x_{n+1} = 2x_n + x_{n-1} $for $ n geq 1.$



Show that: $hspace{2mm} lim_{nto infty} frac{x_n}{x_{n+1}} $ exists.



Then show that the Limit is equal to $sqrt{2}-1$.



For this I thought i could use the fact that $x_n$ is bounded and I thought that it was monotonically falling, but that is not the case, so I ran out of ideas. And I don't know how to calculate the limit... Thank you very much for your help!










share|cite|improve this question
























  • Don´t forget to mark an answer as accepted!!! Have a look at your other questions as well. Then your "$textrm{Thank you very much for your help!}$" is more credible.
    – callculus
    Nov 27 at 21:34

















up vote
3
down vote

favorite












So I have the following problem:
$ x_0 = 1 , x_1 = 2 , $and $x_{n+1} = 2x_n + x_{n-1} $for $ n geq 1.$



Show that: $hspace{2mm} lim_{nto infty} frac{x_n}{x_{n+1}} $ exists.



Then show that the Limit is equal to $sqrt{2}-1$.



For this I thought i could use the fact that $x_n$ is bounded and I thought that it was monotonically falling, but that is not the case, so I ran out of ideas. And I don't know how to calculate the limit... Thank you very much for your help!










share|cite|improve this question
























  • Don´t forget to mark an answer as accepted!!! Have a look at your other questions as well. Then your "$textrm{Thank you very much for your help!}$" is more credible.
    – callculus
    Nov 27 at 21:34















up vote
3
down vote

favorite









up vote
3
down vote

favorite











So I have the following problem:
$ x_0 = 1 , x_1 = 2 , $and $x_{n+1} = 2x_n + x_{n-1} $for $ n geq 1.$



Show that: $hspace{2mm} lim_{nto infty} frac{x_n}{x_{n+1}} $ exists.



Then show that the Limit is equal to $sqrt{2}-1$.



For this I thought i could use the fact that $x_n$ is bounded and I thought that it was monotonically falling, but that is not the case, so I ran out of ideas. And I don't know how to calculate the limit... Thank you very much for your help!










share|cite|improve this question















So I have the following problem:
$ x_0 = 1 , x_1 = 2 , $and $x_{n+1} = 2x_n + x_{n-1} $for $ n geq 1.$



Show that: $hspace{2mm} lim_{nto infty} frac{x_n}{x_{n+1}} $ exists.



Then show that the Limit is equal to $sqrt{2}-1$.



For this I thought i could use the fact that $x_n$ is bounded and I thought that it was monotonically falling, but that is not the case, so I ran out of ideas. And I don't know how to calculate the limit... Thank you very much for your help!







sequences-and-series limits






share|cite|improve this question















share|cite|improve this question













share|cite|improve this question




share|cite|improve this question








edited Nov 26 at 12:04









Tianlalu

2,9011935




2,9011935










asked Nov 26 at 12:03









M-S-R

435




435












  • Don´t forget to mark an answer as accepted!!! Have a look at your other questions as well. Then your "$textrm{Thank you very much for your help!}$" is more credible.
    – callculus
    Nov 27 at 21:34




















  • Don´t forget to mark an answer as accepted!!! Have a look at your other questions as well. Then your "$textrm{Thank you very much for your help!}$" is more credible.
    – callculus
    Nov 27 at 21:34


















Don´t forget to mark an answer as accepted!!! Have a look at your other questions as well. Then your "$textrm{Thank you very much for your help!}$" is more credible.
– callculus
Nov 27 at 21:34






Don´t forget to mark an answer as accepted!!! Have a look at your other questions as well. Then your "$textrm{Thank you very much for your help!}$" is more credible.
– callculus
Nov 27 at 21:34












8 Answers
8






active

oldest

votes

















up vote
9
down vote



accepted










Without guessing the limit you may proceed as follows:




  • Set $q_n = frac{x_n}{x_{n+1}}$
    $$Rightarrow x_{n+1} = 2x_n + x_{n-1} Leftrightarrow frac{1}{q_n} = 2+q_{n-1}Leftrightarrow q_n = frac{1}{2+q_{n-1}}$$


Now, it follows
$$|q_{n+1} - q_n| = left|frac{1}{2+q_{n}} - frac{1}{2+q_{n-1}} right| = left|frac{q_{n-1} - q_n}{(color{blue}{2}+q_{n})(color{blue}{2}+q_{n-1})}right|$$ $$< frac{1}{color{blue}{2cdot 2}}left|q_{n-1} - q_{n} right| stackrel{mbox{see below}}{Rightarrow} boxed{(q_n) mbox{ is convergent}}$$



So, we get for the limit
$$L =frac{1}{2+L} Leftrightarrow (L+1)^2=2 stackrel{L>0}{Rightarrow}boxed{L = sqrt{2}-1}$$



Edit after comment:



Additional info concerning the convergence of the sequence:




  • $q_{n+1} = q_1 + sum_{k=1}^n(q_{k+1} -q_k)$

  • The sums converge (absolutely) as $|q_{k+1} -q_k| < left( frac{1}{4}right)^{k-1}|q_2 - q_1|$ since

  • $left|sum_{k=1}^{infty}(q_{k+1} -q_k) right| leq sum_{k=1}^{infty}|q_{k+1} -q_k| < |q_2 - q_1|sum_{k=1}^{infty} left( frac{1}{4}right)^{k-1} < infty$

  • As $s_n = sum_{k=1}^n(q_{k+1} -q_k)$ converges, it follows that $q_{n+1} =q_1 + s_n$ converges






share|cite|improve this answer























  • I can't understand how you did get the $;<;$ inequality in your center line, and the conclusion that $;q_n;$ is convergent follows but not that immediate: some work there is still due (you can leave it that way but, perhaps, remark that still something must be done)
    – DonAntonio
    Nov 26 at 12:57










  • Nice. Now that inequality is clear...since $;q_n;$ is always positive. The deduction that $;q_n;$ is convergent can now be safely left to the OP (inductive argument). +1
    – DonAntonio
    Nov 26 at 13:14










  • The Edit is fine, yet something more must, imo, be added: there are sequences $;a_n;$ such that $;|a_{n+1}-a_n|to0;$ yet the limit of the sequence does not exist finitely (i.e., the sequence isn't Cauchy). If you leave the above as it is it could mean like you're doing this...
    – DonAntonio
    Nov 26 at 13:26










  • @DonAntonio : Now, the OP has (almost) no work left :-)
    – trancelocation
    Nov 26 at 13:38










  • Oh, he does...yet he asked for help, so at least to remind this. Perhaps something like "From this we can deduce the sequence is Cauchy and..."
    – DonAntonio
    Nov 26 at 13:44




















up vote
4
down vote













We have the general term. With it, the existence and the value for the limit are proven.



For some values of $A$ and $B$ we have.



$$x_n=A(1-sqrt{2})^n+B(1+sqrt{2})^n$$



$$L=lim_{nto+infty}dfrac{x_n}{x_{n+1}}=lim_{nto+infty}dfrac{A(1-sqrt{2})^n+B(1+sqrt{2})^n}{A(1-sqrt{2})^{n+1}+B(1+sqrt{2})^{n+1}}$$



$$L=lim_{nto+infty}dfrac{dfrac{A(1-sqrt{2})^n}{(1+sqrt{2})^n}+dfrac{B(1+sqrt{2})^n}{(1+sqrt{2})^n}}{dfrac{A(1-sqrt{2})^{n+1}}{(1+sqrt{2})^n}+dfrac{B(1+sqrt{2})^{n+1}}{(1+sqrt{2})^n}}$$



$$L=dfrac{0+B}{0+B(1+sqrt{2})}=sqrt{2}-1$$



(the $0$'s come from the exponentials with base less that $1$, as they go to zero as the exponent goes to infinity)



No mind what values for $A$ and $B$ we have, so is, the limit is that, irrespective of the values for $x_1$ and $x_2$.



Added
Suppose the sequence has this form $x_n=A·r^n$ and check if it is possible for $r$ and $A$ to meet the conditions the recurrence law imposes:



$A·r^{n+1}=2A·r^n+A·r^{n-1}$ or $A·r^2r^{n-1}=A2·r·r^{n-1}+Ar^{n-1}$



But obviously $Aneq0$ and $rneq0$, so we can simplify and $r$ must satisfy:



$r^2-2r-1=0$ with roots $1-sqrt{2}$ and $1+sqrt{2}$



But the equation is linear, thus a linear combination of solutions is too a solution:



$x_n=A(1-sqrt{2})^n+B(1+sqrt{2})^n$






share|cite|improve this answer



















  • 1




    How do you know that $A$ and $B$ (not depending on $n$) exist?
    – Kavi Rama Murthy
    Nov 26 at 12:27










  • The linear recurrence laws are always combination of exponentials with the bases the roots of the equation formed with the terms: $r^2-2r-1=0$ in this case.
    – Rafa Budría
    Nov 26 at 12:31








  • 5




    @RafaBudría Kavi, you, and I know that . But it may not be obvious to the question author.
    – Martin R
    Nov 26 at 12:32










  • @MartinR I was thinking of that, but my son, in first course in university is said that property. I supposed it known. Anyway, I can suplement my answer.
    – Rafa Budría
    Nov 26 at 12:34






  • 1




    @YadatiKiran. We get $dfrac{1}{1+sqrt{2}}$, but $dfrac{1}{1+sqrt{2}}=dfrac{sqrt{2}-1}{(1+sqrt{2})(sqrt{2}-1)}=dfrac{sqrt{2}-1}{2-1}=sqrt{2}-1$
    – Rafa Budría
    Nov 26 at 12:55


















up vote
2
down vote













We have $x_n$ to be monotonically increasing and since $dfrac{x_{n+1}}{x_n}=2+dfrac{x_{n-1}}{x_n}$, we can say $dfrac{x_{n+1}}{x_n}<3$ as $dfrac{x_{n-1}}{x_n}<1$ (monotoniclly increasing)$,:forall:ngeq1$. So $x_n$ is convergent.



Let $x_n=Acdot a^n,:aneq0$. The recurrence relation becomes $$ Acdot a^{n+1}=2Acdot a^n+Acdot a^{n-1}implies Acdot a^{n-1}(a^2-2a-1)=0underset{substack{Aneq0\aneq0}}{implies} a^2-2a-1=0$$



By quadratic formula, $a=dfrac{2pmsqrt{4+4}}{2}=1pmsqrt{2}$. Since $x_n$'s are non negative, we have $a=displaystyle lim_{ntoinfty}x_n=1+sqrt{2}$
$rule{17cm}{0.5pt}$



$x_n=A(1+sqrt{2})^n+B(1-sqrt{2})^n$ where $A,B$ are independent of $n$ by linear recurrence.



So if $L=displaystylelim_{ntoinfty}dfrac{x_n}{x_{n+1}}=lim_{ntoinfty}dfrac{A(1+sqrt{2})^n+B(1-sqrt{2})^n}{A(1+sqrt{2})^{n+1}+B(1-sqrt{2})^{n+1}}=lim_{ntoinfty}dfrac{A+dfrac{B(1-sqrt{2})^n}{B(1+sqrt{2})^n}}{dfrac{A(1+sqrt{2})^{n+1}}{(1-sqrt{2})^n}+dfrac{B(1-sqrt{2})^{n+1}}{B(1+sqrt{2})^n}}=dfrac{A}{A(1+sqrt{2})}=sqrt{2}-1$.



$left(text{Since}: |1-sqrt{2}|<1implies displaystylelim_{ntoinfty}(1-sqrt{2})^nto0right)$.



To show that the limit $displaystylelim_{ntoinfty}dfrac{x_n}{x_{n+1}}$ exists we see that it is bounded since $$0<dfrac{x_n}{x_{n+1}}=dfrac{x_n}{2x_{n}+x_{n-1}}leqdfrac{x_n}{2x_{n}}=dfrac12 $$
and $x_{n+1}geq2x_nimplies x_{n+1}geq x_n$ which gives $dfrac{x_n}{x_{n+1}}=dfrac{2x_{n-1}+x_{n-2}}{2x_{n}+x_{n-1}}leqdfrac{2x_{n-1}+x_{n-2}}{2x_{n}}$ i.e.$dfrac{x_n}{x_{n+1}}-dfrac{x_{n-1}}{x_{n}}leqdfrac{x_{n-2}}{2x_{n}}$ which shows monotonicity.Hence the limit exists.






share|cite|improve this answer























  • Wrong. The sequence of ratios is not monotonic. $x_1=1$, $x_2=2$, $x_3=5$, $x_4=12$, $x_5=29$ et cetera. Check for yourself that $x_3/x_2$ is larger than $x_4/x_3$ but also $x_5/x_4$ is larger than $x_4/x_3$. Those ratios jump around the limit approaching it from both sides.
    – Jyrki Lahtonen
    Nov 27 at 4:05










  • @JyrkiLahtonen: I was talking of the sequence ${x_n}$ not the sequence of ratios. I have tried to show the limit of the sequence ${x_n}$ and then tried to estimate the limit of the sequence of ratios.
    – Yadati Kiran
    Nov 27 at 4:19












  • Ok. But $(x_n)$ being monotonic does not imply that $(x_{n-1}/x_n)$ converges. Neither does $(x_n)$, so exactly what are you trying to say in the first paragraph?
    – Jyrki Lahtonen
    Nov 27 at 4:21












  • I never said that. I have assumed that the limit exists but yes I agree I have to show it does. The answer is incomplete.
    – Yadati Kiran
    Nov 27 at 4:25












  • Ok. I do approve of your solution of actually solving the recurrence. That does provide the rigor (if you show that $Aneq0$). Adding my upvote. IMHO your answer would be better without that first paragraph :-)
    – Jyrki Lahtonen
    Nov 27 at 4:30




















up vote
1
down vote













One trick is to use the given limit to derive the existence of it.



Write $y_n=frac{x_n}{x_{n+1}}$.




Claim: There exists $rin (0,1)$ such that
$$(sqrt 2-1)-y_nle rbig(y_{n-1}-(sqrt2-1)big).$$




Proof: Rearranging the terms,
begin{align*}
underbrace{frac{x_{n+1}}{x_n}}_{=1/ {y_n}}-1&=underbrace{frac{x_{n-1}}{x_n}}_{=y_{n-1}}+1\
frac 1{y_n}-(sqrt2+1)&=y_{n-1}-(sqrt2-1)\
frac 1{y_n}-frac1{sqrt2-1}&=y_{n-1}-(sqrt2-1)\
(sqrt 2-1)-y_n&=underbrace{y_n(sqrt2-1)}_{in(0,1)}big(y_{n-1}-(sqrt2-1)big).
end{align*}

so the claim is true. $square$



Therefore,
begin{align*}
|y_n-(sqrt2-1)|&le r |y_{n-1}-(sqrt2-1)|le r^2 |y_{n-2}-(sqrt2-1)|lecdots\
&le r^{n}|y_0-(sqrt2-1)|to0,
end{align*}

implies the limit exists, and $limlimits_{ntoinfty}y_n=sqrt2-1$.






share|cite|improve this answer






























    up vote
    0
    down vote













    Look at $x_n/x_{n+1} - x_{n-1}/x_n$ which is $(x_n^2 - x_{n-1}x_{n+1})/x_n x_{n-1}$.
    We can prove by induction that the numerator is $(-1)^n$.
    $$x_{n+1}^2 - x_n x_{n+2}
    = x_{n+1}^2 - x_n(2x_{n+1} + x_n)
    = x_{n+1}(x_{n+1} - 2x_n) - x_n^2
    = -(x_n^2 - x_{n-1}x_{n+1})$$

    with $x_1^2 - x_0x_2 = -1$. Hence $x_n/x_{n+1}$ tends to a limit by the alternating series test.






    share|cite|improve this answer





















    • You may want to use frac{}{} to write fractions more clearly.
      – DonAntonio
      Nov 26 at 12:58


















    up vote
    0
    down vote













    Since $x_n$ is increasing, all the terms become non-zero. By defining $a_n={x_nover x_{n+1}}$ we have $${x_nover x_{n+1}}={x_nover 2x_n+x_{n-1}}={1over 2+{x_{n-1}over {x_n}}}$$therefore $$a_n={1over 2+{ a_{n-1}}}$$Now by defining $b_n=a_n-(sqrt 2-1)$ we have $$b_n+sqrt 2-1={1over b_{n-1}+sqrt 2+1}$$therefore $$b_n=-sqrt 2+1+{1over b_{n-1}+sqrt 2+1}={(1-sqrt 2)b_{n-1}over a_n+2}$$since $x_n>0$ we have $a_n>0$ therefore$$|b_n|=|{(1-sqrt 2)b_{n-1}over a_n+2}|le {sqrt 2-1over 2}|b_{n-1}|$$which means that $b_n to 0$ or $a_n={x_nover x_{n+1}}to sqrt 2-1 quadblacksquare$






    share|cite|improve this answer




























      up vote
      -1
      down vote













      We have that



      $$frac{x_{n+1}}{x_n} = 2 + frac{x_{n-1}}{x_n}$$



      then by



      $$y_n=frac{x_{n}}{x_{n-1}} implies y_{n+1}=2+frac1{y_n }quad y_0=2$$



      which converges to $L=sqrt 2+1$ and then



      $$lim_{nto infty} frac{x_n}{x_{n+1}}=lim_{nto infty} frac{1}{y_{n+1}}=frac1L=sqrt 2 -1$$






      share|cite|improve this answer



















      • 1




        It seems $y_n$ is 'alternating', i.e. increasing/decreasing when $n$ even and decreasing/increasing when $n$ odd.
        – Tianlalu
        Nov 26 at 13:08






      • 2




        Why $;y_n;$ is increasing and bounded?
        – DonAntonio
        Nov 26 at 13:10










      • @DonAntonio Yes I need to clarify that point better! Thanks
        – gimusi
        Nov 26 at 13:17


















      up vote
      -1
      down vote













      Let



      $$lim_{nto infty} frac{x_n}{x_{n+1}} = lim_{nto infty} frac{x_{n-1}}{x_{n1}} = k$$



      Now



      $$lim_{nto infty} frac{x_n}{x_{n+1}} = k$$
      $$lim_{nto infty} frac{x_n}{2x_{n} + x_{n-1}} = k$$



      Take $x_n$ out from numerator and denominator
      $$lim_{nto infty} frac{1}{2 + frac{x_{n-1}}{x_n}} = k$$



      Using the first equation



      $$ frac{1}{2 + k} = k$$
      $$k^2+2k-1=0$$
      This gives two solutions $k=sqrt{2}-1$ and $k=-sqrt{2}-1$. Since none of the terms can be negative, we reject . the second solution thereby giving us



      $$lim_{nto infty} frac{x_n}{x_{n+1}} = k = sqrt2 - 1$$



      EDIT - As suggested by the commenter we need to prove that it is a finite limit before we start with the proof. Initially it's a $frac{infty}{infty}$ form as both $x_n$ and $x_{n+1}$ approach $infty$ as $n$ approaches $infty$. I'll use a finite upper bound to show that the limit is finite which means it exists.



      For any $n$



      $$frac{x_n}{x_{n+1}} =frac{x_n}{2x_n + x_{n-1}}$$



      As $x_{n-1}$ is always a positive quantity
      $$frac{x_n}{x_{n+1}} leq frac{x_n}{2x_n}$$
      $$frac{x_n}{x_{n+1}} leq frac{1}{2}$$



      For any $n$, you can take the last statement to prove the monotonicity as
      $$x_{n+1}geq x_n$$
      And since both $x_n$ and $x_{n+1}$ are positive values, the lower bound is $0$. The upper bound along with lower bound and the monotonicity proves that the limit is finite.






      share|cite|improve this answer



















      • 3




        This seems to be the less hard part of the question. The hardest part is to prove the limit exists...
        – DonAntonio
        Nov 26 at 12:22






      • 1




        Is there any way to independently do that ?
        – Sauhard Sharma
        Nov 26 at 12:35










      • Why can you write the first line? why is the limit of $frac{x_n}{x_{n+1}} = frac{x_{n-1}}{x_n}$ ?
        – M-S-R
        Nov 26 at 12:54












      • @SauhardSharma It better is, otherwise your whole answer is invalid as you can use arithmetic of limits only if you know that the limit exists finitely, otherwise you can't. You can try induction, for example, to prove monotonicity or something like that
        – DonAntonio
        Nov 26 at 13:02












      • @DonAntonio Thanks for pointing out my mistake. I think the edited proof should suffice
        – Sauhard Sharma
        Nov 26 at 13:18











      Your Answer





      StackExchange.ifUsing("editor", function () {
      return StackExchange.using("mathjaxEditing", function () {
      StackExchange.MarkdownEditor.creationCallbacks.add(function (editor, postfix) {
      StackExchange.mathjaxEditing.prepareWmdForMathJax(editor, postfix, [["$", "$"], ["\\(","\\)"]]);
      });
      });
      }, "mathjax-editing");

      StackExchange.ready(function() {
      var channelOptions = {
      tags: "".split(" "),
      id: "69"
      };
      initTagRenderer("".split(" "), "".split(" "), channelOptions);

      StackExchange.using("externalEditor", function() {
      // Have to fire editor after snippets, if snippets enabled
      if (StackExchange.settings.snippets.snippetsEnabled) {
      StackExchange.using("snippets", function() {
      createEditor();
      });
      }
      else {
      createEditor();
      }
      });

      function createEditor() {
      StackExchange.prepareEditor({
      heartbeatType: 'answer',
      convertImagesToLinks: true,
      noModals: true,
      showLowRepImageUploadWarning: true,
      reputationToPostImages: 10,
      bindNavPrevention: true,
      postfix: "",
      imageUploader: {
      brandingHtml: "Powered by u003ca class="icon-imgur-white" href="https://imgur.com/"u003eu003c/au003e",
      contentPolicyHtml: "User contributions licensed under u003ca href="https://creativecommons.org/licenses/by-sa/3.0/"u003ecc by-sa 3.0 with attribution requiredu003c/au003e u003ca href="https://stackoverflow.com/legal/content-policy"u003e(content policy)u003c/au003e",
      allowUrls: true
      },
      noCode: true, onDemand: true,
      discardSelector: ".discard-answer"
      ,immediatelyShowMarkdownHelp:true
      });


      }
      });














      draft saved

      draft discarded


















      StackExchange.ready(
      function () {
      StackExchange.openid.initPostLogin('.new-post-login', 'https%3a%2f%2fmath.stackexchange.com%2fquestions%2f3014227%2fprove-existence-of-the-limit-of-a-sequence%23new-answer', 'question_page');
      }
      );

      Post as a guest















      Required, but never shown

























      8 Answers
      8






      active

      oldest

      votes








      8 Answers
      8






      active

      oldest

      votes









      active

      oldest

      votes






      active

      oldest

      votes








      up vote
      9
      down vote



      accepted










      Without guessing the limit you may proceed as follows:




      • Set $q_n = frac{x_n}{x_{n+1}}$
        $$Rightarrow x_{n+1} = 2x_n + x_{n-1} Leftrightarrow frac{1}{q_n} = 2+q_{n-1}Leftrightarrow q_n = frac{1}{2+q_{n-1}}$$


      Now, it follows
      $$|q_{n+1} - q_n| = left|frac{1}{2+q_{n}} - frac{1}{2+q_{n-1}} right| = left|frac{q_{n-1} - q_n}{(color{blue}{2}+q_{n})(color{blue}{2}+q_{n-1})}right|$$ $$< frac{1}{color{blue}{2cdot 2}}left|q_{n-1} - q_{n} right| stackrel{mbox{see below}}{Rightarrow} boxed{(q_n) mbox{ is convergent}}$$



      So, we get for the limit
      $$L =frac{1}{2+L} Leftrightarrow (L+1)^2=2 stackrel{L>0}{Rightarrow}boxed{L = sqrt{2}-1}$$



      Edit after comment:



      Additional info concerning the convergence of the sequence:




      • $q_{n+1} = q_1 + sum_{k=1}^n(q_{k+1} -q_k)$

      • The sums converge (absolutely) as $|q_{k+1} -q_k| < left( frac{1}{4}right)^{k-1}|q_2 - q_1|$ since

      • $left|sum_{k=1}^{infty}(q_{k+1} -q_k) right| leq sum_{k=1}^{infty}|q_{k+1} -q_k| < |q_2 - q_1|sum_{k=1}^{infty} left( frac{1}{4}right)^{k-1} < infty$

      • As $s_n = sum_{k=1}^n(q_{k+1} -q_k)$ converges, it follows that $q_{n+1} =q_1 + s_n$ converges






      share|cite|improve this answer























      • I can't understand how you did get the $;<;$ inequality in your center line, and the conclusion that $;q_n;$ is convergent follows but not that immediate: some work there is still due (you can leave it that way but, perhaps, remark that still something must be done)
        – DonAntonio
        Nov 26 at 12:57










      • Nice. Now that inequality is clear...since $;q_n;$ is always positive. The deduction that $;q_n;$ is convergent can now be safely left to the OP (inductive argument). +1
        – DonAntonio
        Nov 26 at 13:14










      • The Edit is fine, yet something more must, imo, be added: there are sequences $;a_n;$ such that $;|a_{n+1}-a_n|to0;$ yet the limit of the sequence does not exist finitely (i.e., the sequence isn't Cauchy). If you leave the above as it is it could mean like you're doing this...
        – DonAntonio
        Nov 26 at 13:26










      • @DonAntonio : Now, the OP has (almost) no work left :-)
        – trancelocation
        Nov 26 at 13:38










      • Oh, he does...yet he asked for help, so at least to remind this. Perhaps something like "From this we can deduce the sequence is Cauchy and..."
        – DonAntonio
        Nov 26 at 13:44

















      up vote
      9
      down vote



      accepted










      Without guessing the limit you may proceed as follows:




      • Set $q_n = frac{x_n}{x_{n+1}}$
        $$Rightarrow x_{n+1} = 2x_n + x_{n-1} Leftrightarrow frac{1}{q_n} = 2+q_{n-1}Leftrightarrow q_n = frac{1}{2+q_{n-1}}$$


      Now, it follows
      $$|q_{n+1} - q_n| = left|frac{1}{2+q_{n}} - frac{1}{2+q_{n-1}} right| = left|frac{q_{n-1} - q_n}{(color{blue}{2}+q_{n})(color{blue}{2}+q_{n-1})}right|$$ $$< frac{1}{color{blue}{2cdot 2}}left|q_{n-1} - q_{n} right| stackrel{mbox{see below}}{Rightarrow} boxed{(q_n) mbox{ is convergent}}$$



      So, we get for the limit
      $$L =frac{1}{2+L} Leftrightarrow (L+1)^2=2 stackrel{L>0}{Rightarrow}boxed{L = sqrt{2}-1}$$



      Edit after comment:



      Additional info concerning the convergence of the sequence:




      • $q_{n+1} = q_1 + sum_{k=1}^n(q_{k+1} -q_k)$

      • The sums converge (absolutely) as $|q_{k+1} -q_k| < left( frac{1}{4}right)^{k-1}|q_2 - q_1|$ since

      • $left|sum_{k=1}^{infty}(q_{k+1} -q_k) right| leq sum_{k=1}^{infty}|q_{k+1} -q_k| < |q_2 - q_1|sum_{k=1}^{infty} left( frac{1}{4}right)^{k-1} < infty$

      • As $s_n = sum_{k=1}^n(q_{k+1} -q_k)$ converges, it follows that $q_{n+1} =q_1 + s_n$ converges






      share|cite|improve this answer























      • I can't understand how you did get the $;<;$ inequality in your center line, and the conclusion that $;q_n;$ is convergent follows but not that immediate: some work there is still due (you can leave it that way but, perhaps, remark that still something must be done)
        – DonAntonio
        Nov 26 at 12:57










      • Nice. Now that inequality is clear...since $;q_n;$ is always positive. The deduction that $;q_n;$ is convergent can now be safely left to the OP (inductive argument). +1
        – DonAntonio
        Nov 26 at 13:14










      • The Edit is fine, yet something more must, imo, be added: there are sequences $;a_n;$ such that $;|a_{n+1}-a_n|to0;$ yet the limit of the sequence does not exist finitely (i.e., the sequence isn't Cauchy). If you leave the above as it is it could mean like you're doing this...
        – DonAntonio
        Nov 26 at 13:26










      • @DonAntonio : Now, the OP has (almost) no work left :-)
        – trancelocation
        Nov 26 at 13:38










      • Oh, he does...yet he asked for help, so at least to remind this. Perhaps something like "From this we can deduce the sequence is Cauchy and..."
        – DonAntonio
        Nov 26 at 13:44















      up vote
      9
      down vote



      accepted







      up vote
      9
      down vote



      accepted






      Without guessing the limit you may proceed as follows:




      • Set $q_n = frac{x_n}{x_{n+1}}$
        $$Rightarrow x_{n+1} = 2x_n + x_{n-1} Leftrightarrow frac{1}{q_n} = 2+q_{n-1}Leftrightarrow q_n = frac{1}{2+q_{n-1}}$$


      Now, it follows
      $$|q_{n+1} - q_n| = left|frac{1}{2+q_{n}} - frac{1}{2+q_{n-1}} right| = left|frac{q_{n-1} - q_n}{(color{blue}{2}+q_{n})(color{blue}{2}+q_{n-1})}right|$$ $$< frac{1}{color{blue}{2cdot 2}}left|q_{n-1} - q_{n} right| stackrel{mbox{see below}}{Rightarrow} boxed{(q_n) mbox{ is convergent}}$$



      So, we get for the limit
      $$L =frac{1}{2+L} Leftrightarrow (L+1)^2=2 stackrel{L>0}{Rightarrow}boxed{L = sqrt{2}-1}$$



      Edit after comment:



      Additional info concerning the convergence of the sequence:




      • $q_{n+1} = q_1 + sum_{k=1}^n(q_{k+1} -q_k)$

      • The sums converge (absolutely) as $|q_{k+1} -q_k| < left( frac{1}{4}right)^{k-1}|q_2 - q_1|$ since

      • $left|sum_{k=1}^{infty}(q_{k+1} -q_k) right| leq sum_{k=1}^{infty}|q_{k+1} -q_k| < |q_2 - q_1|sum_{k=1}^{infty} left( frac{1}{4}right)^{k-1} < infty$

      • As $s_n = sum_{k=1}^n(q_{k+1} -q_k)$ converges, it follows that $q_{n+1} =q_1 + s_n$ converges






      share|cite|improve this answer














      Without guessing the limit you may proceed as follows:




      • Set $q_n = frac{x_n}{x_{n+1}}$
        $$Rightarrow x_{n+1} = 2x_n + x_{n-1} Leftrightarrow frac{1}{q_n} = 2+q_{n-1}Leftrightarrow q_n = frac{1}{2+q_{n-1}}$$


      Now, it follows
      $$|q_{n+1} - q_n| = left|frac{1}{2+q_{n}} - frac{1}{2+q_{n-1}} right| = left|frac{q_{n-1} - q_n}{(color{blue}{2}+q_{n})(color{blue}{2}+q_{n-1})}right|$$ $$< frac{1}{color{blue}{2cdot 2}}left|q_{n-1} - q_{n} right| stackrel{mbox{see below}}{Rightarrow} boxed{(q_n) mbox{ is convergent}}$$



      So, we get for the limit
      $$L =frac{1}{2+L} Leftrightarrow (L+1)^2=2 stackrel{L>0}{Rightarrow}boxed{L = sqrt{2}-1}$$



      Edit after comment:



      Additional info concerning the convergence of the sequence:




      • $q_{n+1} = q_1 + sum_{k=1}^n(q_{k+1} -q_k)$

      • The sums converge (absolutely) as $|q_{k+1} -q_k| < left( frac{1}{4}right)^{k-1}|q_2 - q_1|$ since

      • $left|sum_{k=1}^{infty}(q_{k+1} -q_k) right| leq sum_{k=1}^{infty}|q_{k+1} -q_k| < |q_2 - q_1|sum_{k=1}^{infty} left( frac{1}{4}right)^{k-1} < infty$

      • As $s_n = sum_{k=1}^n(q_{k+1} -q_k)$ converges, it follows that $q_{n+1} =q_1 + s_n$ converges







      share|cite|improve this answer














      share|cite|improve this answer



      share|cite|improve this answer








      edited Nov 26 at 13:36

























      answered Nov 26 at 12:44









      trancelocation

      8,5821520




      8,5821520












      • I can't understand how you did get the $;<;$ inequality in your center line, and the conclusion that $;q_n;$ is convergent follows but not that immediate: some work there is still due (you can leave it that way but, perhaps, remark that still something must be done)
        – DonAntonio
        Nov 26 at 12:57










      • Nice. Now that inequality is clear...since $;q_n;$ is always positive. The deduction that $;q_n;$ is convergent can now be safely left to the OP (inductive argument). +1
        – DonAntonio
        Nov 26 at 13:14










      • The Edit is fine, yet something more must, imo, be added: there are sequences $;a_n;$ such that $;|a_{n+1}-a_n|to0;$ yet the limit of the sequence does not exist finitely (i.e., the sequence isn't Cauchy). If you leave the above as it is it could mean like you're doing this...
        – DonAntonio
        Nov 26 at 13:26










      • @DonAntonio : Now, the OP has (almost) no work left :-)
        – trancelocation
        Nov 26 at 13:38










      • Oh, he does...yet he asked for help, so at least to remind this. Perhaps something like "From this we can deduce the sequence is Cauchy and..."
        – DonAntonio
        Nov 26 at 13:44




















      • I can't understand how you did get the $;<;$ inequality in your center line, and the conclusion that $;q_n;$ is convergent follows but not that immediate: some work there is still due (you can leave it that way but, perhaps, remark that still something must be done)
        – DonAntonio
        Nov 26 at 12:57










      • Nice. Now that inequality is clear...since $;q_n;$ is always positive. The deduction that $;q_n;$ is convergent can now be safely left to the OP (inductive argument). +1
        – DonAntonio
        Nov 26 at 13:14










      • The Edit is fine, yet something more must, imo, be added: there are sequences $;a_n;$ such that $;|a_{n+1}-a_n|to0;$ yet the limit of the sequence does not exist finitely (i.e., the sequence isn't Cauchy). If you leave the above as it is it could mean like you're doing this...
        – DonAntonio
        Nov 26 at 13:26










      • @DonAntonio : Now, the OP has (almost) no work left :-)
        – trancelocation
        Nov 26 at 13:38










      • Oh, he does...yet he asked for help, so at least to remind this. Perhaps something like "From this we can deduce the sequence is Cauchy and..."
        – DonAntonio
        Nov 26 at 13:44


















      I can't understand how you did get the $;<;$ inequality in your center line, and the conclusion that $;q_n;$ is convergent follows but not that immediate: some work there is still due (you can leave it that way but, perhaps, remark that still something must be done)
      – DonAntonio
      Nov 26 at 12:57




      I can't understand how you did get the $;<;$ inequality in your center line, and the conclusion that $;q_n;$ is convergent follows but not that immediate: some work there is still due (you can leave it that way but, perhaps, remark that still something must be done)
      – DonAntonio
      Nov 26 at 12:57












      Nice. Now that inequality is clear...since $;q_n;$ is always positive. The deduction that $;q_n;$ is convergent can now be safely left to the OP (inductive argument). +1
      – DonAntonio
      Nov 26 at 13:14




      Nice. Now that inequality is clear...since $;q_n;$ is always positive. The deduction that $;q_n;$ is convergent can now be safely left to the OP (inductive argument). +1
      – DonAntonio
      Nov 26 at 13:14












      The Edit is fine, yet something more must, imo, be added: there are sequences $;a_n;$ such that $;|a_{n+1}-a_n|to0;$ yet the limit of the sequence does not exist finitely (i.e., the sequence isn't Cauchy). If you leave the above as it is it could mean like you're doing this...
      – DonAntonio
      Nov 26 at 13:26




      The Edit is fine, yet something more must, imo, be added: there are sequences $;a_n;$ such that $;|a_{n+1}-a_n|to0;$ yet the limit of the sequence does not exist finitely (i.e., the sequence isn't Cauchy). If you leave the above as it is it could mean like you're doing this...
      – DonAntonio
      Nov 26 at 13:26












      @DonAntonio : Now, the OP has (almost) no work left :-)
      – trancelocation
      Nov 26 at 13:38




      @DonAntonio : Now, the OP has (almost) no work left :-)
      – trancelocation
      Nov 26 at 13:38












      Oh, he does...yet he asked for help, so at least to remind this. Perhaps something like "From this we can deduce the sequence is Cauchy and..."
      – DonAntonio
      Nov 26 at 13:44






      Oh, he does...yet he asked for help, so at least to remind this. Perhaps something like "From this we can deduce the sequence is Cauchy and..."
      – DonAntonio
      Nov 26 at 13:44












      up vote
      4
      down vote













      We have the general term. With it, the existence and the value for the limit are proven.



      For some values of $A$ and $B$ we have.



      $$x_n=A(1-sqrt{2})^n+B(1+sqrt{2})^n$$



      $$L=lim_{nto+infty}dfrac{x_n}{x_{n+1}}=lim_{nto+infty}dfrac{A(1-sqrt{2})^n+B(1+sqrt{2})^n}{A(1-sqrt{2})^{n+1}+B(1+sqrt{2})^{n+1}}$$



      $$L=lim_{nto+infty}dfrac{dfrac{A(1-sqrt{2})^n}{(1+sqrt{2})^n}+dfrac{B(1+sqrt{2})^n}{(1+sqrt{2})^n}}{dfrac{A(1-sqrt{2})^{n+1}}{(1+sqrt{2})^n}+dfrac{B(1+sqrt{2})^{n+1}}{(1+sqrt{2})^n}}$$



      $$L=dfrac{0+B}{0+B(1+sqrt{2})}=sqrt{2}-1$$



      (the $0$'s come from the exponentials with base less that $1$, as they go to zero as the exponent goes to infinity)



      No mind what values for $A$ and $B$ we have, so is, the limit is that, irrespective of the values for $x_1$ and $x_2$.



      Added
      Suppose the sequence has this form $x_n=A·r^n$ and check if it is possible for $r$ and $A$ to meet the conditions the recurrence law imposes:



      $A·r^{n+1}=2A·r^n+A·r^{n-1}$ or $A·r^2r^{n-1}=A2·r·r^{n-1}+Ar^{n-1}$



      But obviously $Aneq0$ and $rneq0$, so we can simplify and $r$ must satisfy:



      $r^2-2r-1=0$ with roots $1-sqrt{2}$ and $1+sqrt{2}$



      But the equation is linear, thus a linear combination of solutions is too a solution:



      $x_n=A(1-sqrt{2})^n+B(1+sqrt{2})^n$






      share|cite|improve this answer



















      • 1




        How do you know that $A$ and $B$ (not depending on $n$) exist?
        – Kavi Rama Murthy
        Nov 26 at 12:27










      • The linear recurrence laws are always combination of exponentials with the bases the roots of the equation formed with the terms: $r^2-2r-1=0$ in this case.
        – Rafa Budría
        Nov 26 at 12:31








      • 5




        @RafaBudría Kavi, you, and I know that . But it may not be obvious to the question author.
        – Martin R
        Nov 26 at 12:32










      • @MartinR I was thinking of that, but my son, in first course in university is said that property. I supposed it known. Anyway, I can suplement my answer.
        – Rafa Budría
        Nov 26 at 12:34






      • 1




        @YadatiKiran. We get $dfrac{1}{1+sqrt{2}}$, but $dfrac{1}{1+sqrt{2}}=dfrac{sqrt{2}-1}{(1+sqrt{2})(sqrt{2}-1)}=dfrac{sqrt{2}-1}{2-1}=sqrt{2}-1$
        – Rafa Budría
        Nov 26 at 12:55















      up vote
      4
      down vote













      We have the general term. With it, the existence and the value for the limit are proven.



      For some values of $A$ and $B$ we have.



      $$x_n=A(1-sqrt{2})^n+B(1+sqrt{2})^n$$



      $$L=lim_{nto+infty}dfrac{x_n}{x_{n+1}}=lim_{nto+infty}dfrac{A(1-sqrt{2})^n+B(1+sqrt{2})^n}{A(1-sqrt{2})^{n+1}+B(1+sqrt{2})^{n+1}}$$



      $$L=lim_{nto+infty}dfrac{dfrac{A(1-sqrt{2})^n}{(1+sqrt{2})^n}+dfrac{B(1+sqrt{2})^n}{(1+sqrt{2})^n}}{dfrac{A(1-sqrt{2})^{n+1}}{(1+sqrt{2})^n}+dfrac{B(1+sqrt{2})^{n+1}}{(1+sqrt{2})^n}}$$



      $$L=dfrac{0+B}{0+B(1+sqrt{2})}=sqrt{2}-1$$



      (the $0$'s come from the exponentials with base less that $1$, as they go to zero as the exponent goes to infinity)



      No mind what values for $A$ and $B$ we have, so is, the limit is that, irrespective of the values for $x_1$ and $x_2$.



      Added
      Suppose the sequence has this form $x_n=A·r^n$ and check if it is possible for $r$ and $A$ to meet the conditions the recurrence law imposes:



      $A·r^{n+1}=2A·r^n+A·r^{n-1}$ or $A·r^2r^{n-1}=A2·r·r^{n-1}+Ar^{n-1}$



      But obviously $Aneq0$ and $rneq0$, so we can simplify and $r$ must satisfy:



      $r^2-2r-1=0$ with roots $1-sqrt{2}$ and $1+sqrt{2}$



      But the equation is linear, thus a linear combination of solutions is too a solution:



      $x_n=A(1-sqrt{2})^n+B(1+sqrt{2})^n$






      share|cite|improve this answer



















      • 1




        How do you know that $A$ and $B$ (not depending on $n$) exist?
        – Kavi Rama Murthy
        Nov 26 at 12:27










      • The linear recurrence laws are always combination of exponentials with the bases the roots of the equation formed with the terms: $r^2-2r-1=0$ in this case.
        – Rafa Budría
        Nov 26 at 12:31








      • 5




        @RafaBudría Kavi, you, and I know that . But it may not be obvious to the question author.
        – Martin R
        Nov 26 at 12:32










      • @MartinR I was thinking of that, but my son, in first course in university is said that property. I supposed it known. Anyway, I can suplement my answer.
        – Rafa Budría
        Nov 26 at 12:34






      • 1




        @YadatiKiran. We get $dfrac{1}{1+sqrt{2}}$, but $dfrac{1}{1+sqrt{2}}=dfrac{sqrt{2}-1}{(1+sqrt{2})(sqrt{2}-1)}=dfrac{sqrt{2}-1}{2-1}=sqrt{2}-1$
        – Rafa Budría
        Nov 26 at 12:55













      up vote
      4
      down vote










      up vote
      4
      down vote









      We have the general term. With it, the existence and the value for the limit are proven.



      For some values of $A$ and $B$ we have.



      $$x_n=A(1-sqrt{2})^n+B(1+sqrt{2})^n$$



      $$L=lim_{nto+infty}dfrac{x_n}{x_{n+1}}=lim_{nto+infty}dfrac{A(1-sqrt{2})^n+B(1+sqrt{2})^n}{A(1-sqrt{2})^{n+1}+B(1+sqrt{2})^{n+1}}$$



      $$L=lim_{nto+infty}dfrac{dfrac{A(1-sqrt{2})^n}{(1+sqrt{2})^n}+dfrac{B(1+sqrt{2})^n}{(1+sqrt{2})^n}}{dfrac{A(1-sqrt{2})^{n+1}}{(1+sqrt{2})^n}+dfrac{B(1+sqrt{2})^{n+1}}{(1+sqrt{2})^n}}$$



      $$L=dfrac{0+B}{0+B(1+sqrt{2})}=sqrt{2}-1$$



      (the $0$'s come from the exponentials with base less that $1$, as they go to zero as the exponent goes to infinity)



      No mind what values for $A$ and $B$ we have, so is, the limit is that, irrespective of the values for $x_1$ and $x_2$.



      Added
      Suppose the sequence has this form $x_n=A·r^n$ and check if it is possible for $r$ and $A$ to meet the conditions the recurrence law imposes:



      $A·r^{n+1}=2A·r^n+A·r^{n-1}$ or $A·r^2r^{n-1}=A2·r·r^{n-1}+Ar^{n-1}$



      But obviously $Aneq0$ and $rneq0$, so we can simplify and $r$ must satisfy:



      $r^2-2r-1=0$ with roots $1-sqrt{2}$ and $1+sqrt{2}$



      But the equation is linear, thus a linear combination of solutions is too a solution:



      $x_n=A(1-sqrt{2})^n+B(1+sqrt{2})^n$






      share|cite|improve this answer














      We have the general term. With it, the existence and the value for the limit are proven.



      For some values of $A$ and $B$ we have.



      $$x_n=A(1-sqrt{2})^n+B(1+sqrt{2})^n$$



      $$L=lim_{nto+infty}dfrac{x_n}{x_{n+1}}=lim_{nto+infty}dfrac{A(1-sqrt{2})^n+B(1+sqrt{2})^n}{A(1-sqrt{2})^{n+1}+B(1+sqrt{2})^{n+1}}$$



      $$L=lim_{nto+infty}dfrac{dfrac{A(1-sqrt{2})^n}{(1+sqrt{2})^n}+dfrac{B(1+sqrt{2})^n}{(1+sqrt{2})^n}}{dfrac{A(1-sqrt{2})^{n+1}}{(1+sqrt{2})^n}+dfrac{B(1+sqrt{2})^{n+1}}{(1+sqrt{2})^n}}$$



      $$L=dfrac{0+B}{0+B(1+sqrt{2})}=sqrt{2}-1$$



      (the $0$'s come from the exponentials with base less that $1$, as they go to zero as the exponent goes to infinity)



      No mind what values for $A$ and $B$ we have, so is, the limit is that, irrespective of the values for $x_1$ and $x_2$.



      Added
      Suppose the sequence has this form $x_n=A·r^n$ and check if it is possible for $r$ and $A$ to meet the conditions the recurrence law imposes:



      $A·r^{n+1}=2A·r^n+A·r^{n-1}$ or $A·r^2r^{n-1}=A2·r·r^{n-1}+Ar^{n-1}$



      But obviously $Aneq0$ and $rneq0$, so we can simplify and $r$ must satisfy:



      $r^2-2r-1=0$ with roots $1-sqrt{2}$ and $1+sqrt{2}$



      But the equation is linear, thus a linear combination of solutions is too a solution:



      $x_n=A(1-sqrt{2})^n+B(1+sqrt{2})^n$







      share|cite|improve this answer














      share|cite|improve this answer



      share|cite|improve this answer








      edited Nov 26 at 12:47

























      answered Nov 26 at 12:24









      Rafa Budría

      5,4001825




      5,4001825








      • 1




        How do you know that $A$ and $B$ (not depending on $n$) exist?
        – Kavi Rama Murthy
        Nov 26 at 12:27










      • The linear recurrence laws are always combination of exponentials with the bases the roots of the equation formed with the terms: $r^2-2r-1=0$ in this case.
        – Rafa Budría
        Nov 26 at 12:31








      • 5




        @RafaBudría Kavi, you, and I know that . But it may not be obvious to the question author.
        – Martin R
        Nov 26 at 12:32










      • @MartinR I was thinking of that, but my son, in first course in university is said that property. I supposed it known. Anyway, I can suplement my answer.
        – Rafa Budría
        Nov 26 at 12:34






      • 1




        @YadatiKiran. We get $dfrac{1}{1+sqrt{2}}$, but $dfrac{1}{1+sqrt{2}}=dfrac{sqrt{2}-1}{(1+sqrt{2})(sqrt{2}-1)}=dfrac{sqrt{2}-1}{2-1}=sqrt{2}-1$
        – Rafa Budría
        Nov 26 at 12:55














      • 1




        How do you know that $A$ and $B$ (not depending on $n$) exist?
        – Kavi Rama Murthy
        Nov 26 at 12:27










      • The linear recurrence laws are always combination of exponentials with the bases the roots of the equation formed with the terms: $r^2-2r-1=0$ in this case.
        – Rafa Budría
        Nov 26 at 12:31








      • 5




        @RafaBudría Kavi, you, and I know that . But it may not be obvious to the question author.
        – Martin R
        Nov 26 at 12:32










      • @MartinR I was thinking of that, but my son, in first course in university is said that property. I supposed it known. Anyway, I can suplement my answer.
        – Rafa Budría
        Nov 26 at 12:34






      • 1




        @YadatiKiran. We get $dfrac{1}{1+sqrt{2}}$, but $dfrac{1}{1+sqrt{2}}=dfrac{sqrt{2}-1}{(1+sqrt{2})(sqrt{2}-1)}=dfrac{sqrt{2}-1}{2-1}=sqrt{2}-1$
        – Rafa Budría
        Nov 26 at 12:55








      1




      1




      How do you know that $A$ and $B$ (not depending on $n$) exist?
      – Kavi Rama Murthy
      Nov 26 at 12:27




      How do you know that $A$ and $B$ (not depending on $n$) exist?
      – Kavi Rama Murthy
      Nov 26 at 12:27












      The linear recurrence laws are always combination of exponentials with the bases the roots of the equation formed with the terms: $r^2-2r-1=0$ in this case.
      – Rafa Budría
      Nov 26 at 12:31






      The linear recurrence laws are always combination of exponentials with the bases the roots of the equation formed with the terms: $r^2-2r-1=0$ in this case.
      – Rafa Budría
      Nov 26 at 12:31






      5




      5




      @RafaBudría Kavi, you, and I know that . But it may not be obvious to the question author.
      – Martin R
      Nov 26 at 12:32




      @RafaBudría Kavi, you, and I know that . But it may not be obvious to the question author.
      – Martin R
      Nov 26 at 12:32












      @MartinR I was thinking of that, but my son, in first course in university is said that property. I supposed it known. Anyway, I can suplement my answer.
      – Rafa Budría
      Nov 26 at 12:34




      @MartinR I was thinking of that, but my son, in first course in university is said that property. I supposed it known. Anyway, I can suplement my answer.
      – Rafa Budría
      Nov 26 at 12:34




      1




      1




      @YadatiKiran. We get $dfrac{1}{1+sqrt{2}}$, but $dfrac{1}{1+sqrt{2}}=dfrac{sqrt{2}-1}{(1+sqrt{2})(sqrt{2}-1)}=dfrac{sqrt{2}-1}{2-1}=sqrt{2}-1$
      – Rafa Budría
      Nov 26 at 12:55




      @YadatiKiran. We get $dfrac{1}{1+sqrt{2}}$, but $dfrac{1}{1+sqrt{2}}=dfrac{sqrt{2}-1}{(1+sqrt{2})(sqrt{2}-1)}=dfrac{sqrt{2}-1}{2-1}=sqrt{2}-1$
      – Rafa Budría
      Nov 26 at 12:55










      up vote
      2
      down vote













      We have $x_n$ to be monotonically increasing and since $dfrac{x_{n+1}}{x_n}=2+dfrac{x_{n-1}}{x_n}$, we can say $dfrac{x_{n+1}}{x_n}<3$ as $dfrac{x_{n-1}}{x_n}<1$ (monotoniclly increasing)$,:forall:ngeq1$. So $x_n$ is convergent.



      Let $x_n=Acdot a^n,:aneq0$. The recurrence relation becomes $$ Acdot a^{n+1}=2Acdot a^n+Acdot a^{n-1}implies Acdot a^{n-1}(a^2-2a-1)=0underset{substack{Aneq0\aneq0}}{implies} a^2-2a-1=0$$



      By quadratic formula, $a=dfrac{2pmsqrt{4+4}}{2}=1pmsqrt{2}$. Since $x_n$'s are non negative, we have $a=displaystyle lim_{ntoinfty}x_n=1+sqrt{2}$
      $rule{17cm}{0.5pt}$



      $x_n=A(1+sqrt{2})^n+B(1-sqrt{2})^n$ where $A,B$ are independent of $n$ by linear recurrence.



      So if $L=displaystylelim_{ntoinfty}dfrac{x_n}{x_{n+1}}=lim_{ntoinfty}dfrac{A(1+sqrt{2})^n+B(1-sqrt{2})^n}{A(1+sqrt{2})^{n+1}+B(1-sqrt{2})^{n+1}}=lim_{ntoinfty}dfrac{A+dfrac{B(1-sqrt{2})^n}{B(1+sqrt{2})^n}}{dfrac{A(1+sqrt{2})^{n+1}}{(1-sqrt{2})^n}+dfrac{B(1-sqrt{2})^{n+1}}{B(1+sqrt{2})^n}}=dfrac{A}{A(1+sqrt{2})}=sqrt{2}-1$.



      $left(text{Since}: |1-sqrt{2}|<1implies displaystylelim_{ntoinfty}(1-sqrt{2})^nto0right)$.



      To show that the limit $displaystylelim_{ntoinfty}dfrac{x_n}{x_{n+1}}$ exists we see that it is bounded since $$0<dfrac{x_n}{x_{n+1}}=dfrac{x_n}{2x_{n}+x_{n-1}}leqdfrac{x_n}{2x_{n}}=dfrac12 $$
      and $x_{n+1}geq2x_nimplies x_{n+1}geq x_n$ which gives $dfrac{x_n}{x_{n+1}}=dfrac{2x_{n-1}+x_{n-2}}{2x_{n}+x_{n-1}}leqdfrac{2x_{n-1}+x_{n-2}}{2x_{n}}$ i.e.$dfrac{x_n}{x_{n+1}}-dfrac{x_{n-1}}{x_{n}}leqdfrac{x_{n-2}}{2x_{n}}$ which shows monotonicity.Hence the limit exists.






      share|cite|improve this answer























      • Wrong. The sequence of ratios is not monotonic. $x_1=1$, $x_2=2$, $x_3=5$, $x_4=12$, $x_5=29$ et cetera. Check for yourself that $x_3/x_2$ is larger than $x_4/x_3$ but also $x_5/x_4$ is larger than $x_4/x_3$. Those ratios jump around the limit approaching it from both sides.
        – Jyrki Lahtonen
        Nov 27 at 4:05










      • @JyrkiLahtonen: I was talking of the sequence ${x_n}$ not the sequence of ratios. I have tried to show the limit of the sequence ${x_n}$ and then tried to estimate the limit of the sequence of ratios.
        – Yadati Kiran
        Nov 27 at 4:19












      • Ok. But $(x_n)$ being monotonic does not imply that $(x_{n-1}/x_n)$ converges. Neither does $(x_n)$, so exactly what are you trying to say in the first paragraph?
        – Jyrki Lahtonen
        Nov 27 at 4:21












      • I never said that. I have assumed that the limit exists but yes I agree I have to show it does. The answer is incomplete.
        – Yadati Kiran
        Nov 27 at 4:25












      • Ok. I do approve of your solution of actually solving the recurrence. That does provide the rigor (if you show that $Aneq0$). Adding my upvote. IMHO your answer would be better without that first paragraph :-)
        – Jyrki Lahtonen
        Nov 27 at 4:30

















      up vote
      2
      down vote













      We have $x_n$ to be monotonically increasing and since $dfrac{x_{n+1}}{x_n}=2+dfrac{x_{n-1}}{x_n}$, we can say $dfrac{x_{n+1}}{x_n}<3$ as $dfrac{x_{n-1}}{x_n}<1$ (monotoniclly increasing)$,:forall:ngeq1$. So $x_n$ is convergent.



      Let $x_n=Acdot a^n,:aneq0$. The recurrence relation becomes $$ Acdot a^{n+1}=2Acdot a^n+Acdot a^{n-1}implies Acdot a^{n-1}(a^2-2a-1)=0underset{substack{Aneq0\aneq0}}{implies} a^2-2a-1=0$$



      By quadratic formula, $a=dfrac{2pmsqrt{4+4}}{2}=1pmsqrt{2}$. Since $x_n$'s are non negative, we have $a=displaystyle lim_{ntoinfty}x_n=1+sqrt{2}$
      $rule{17cm}{0.5pt}$



      $x_n=A(1+sqrt{2})^n+B(1-sqrt{2})^n$ where $A,B$ are independent of $n$ by linear recurrence.



      So if $L=displaystylelim_{ntoinfty}dfrac{x_n}{x_{n+1}}=lim_{ntoinfty}dfrac{A(1+sqrt{2})^n+B(1-sqrt{2})^n}{A(1+sqrt{2})^{n+1}+B(1-sqrt{2})^{n+1}}=lim_{ntoinfty}dfrac{A+dfrac{B(1-sqrt{2})^n}{B(1+sqrt{2})^n}}{dfrac{A(1+sqrt{2})^{n+1}}{(1-sqrt{2})^n}+dfrac{B(1-sqrt{2})^{n+1}}{B(1+sqrt{2})^n}}=dfrac{A}{A(1+sqrt{2})}=sqrt{2}-1$.



      $left(text{Since}: |1-sqrt{2}|<1implies displaystylelim_{ntoinfty}(1-sqrt{2})^nto0right)$.



      To show that the limit $displaystylelim_{ntoinfty}dfrac{x_n}{x_{n+1}}$ exists we see that it is bounded since $$0<dfrac{x_n}{x_{n+1}}=dfrac{x_n}{2x_{n}+x_{n-1}}leqdfrac{x_n}{2x_{n}}=dfrac12 $$
      and $x_{n+1}geq2x_nimplies x_{n+1}geq x_n$ which gives $dfrac{x_n}{x_{n+1}}=dfrac{2x_{n-1}+x_{n-2}}{2x_{n}+x_{n-1}}leqdfrac{2x_{n-1}+x_{n-2}}{2x_{n}}$ i.e.$dfrac{x_n}{x_{n+1}}-dfrac{x_{n-1}}{x_{n}}leqdfrac{x_{n-2}}{2x_{n}}$ which shows monotonicity.Hence the limit exists.






      share|cite|improve this answer























      • Wrong. The sequence of ratios is not monotonic. $x_1=1$, $x_2=2$, $x_3=5$, $x_4=12$, $x_5=29$ et cetera. Check for yourself that $x_3/x_2$ is larger than $x_4/x_3$ but also $x_5/x_4$ is larger than $x_4/x_3$. Those ratios jump around the limit approaching it from both sides.
        – Jyrki Lahtonen
        Nov 27 at 4:05










      • @JyrkiLahtonen: I was talking of the sequence ${x_n}$ not the sequence of ratios. I have tried to show the limit of the sequence ${x_n}$ and then tried to estimate the limit of the sequence of ratios.
        – Yadati Kiran
        Nov 27 at 4:19












      • Ok. But $(x_n)$ being monotonic does not imply that $(x_{n-1}/x_n)$ converges. Neither does $(x_n)$, so exactly what are you trying to say in the first paragraph?
        – Jyrki Lahtonen
        Nov 27 at 4:21












      • I never said that. I have assumed that the limit exists but yes I agree I have to show it does. The answer is incomplete.
        – Yadati Kiran
        Nov 27 at 4:25












      • Ok. I do approve of your solution of actually solving the recurrence. That does provide the rigor (if you show that $Aneq0$). Adding my upvote. IMHO your answer would be better without that first paragraph :-)
        – Jyrki Lahtonen
        Nov 27 at 4:30















      up vote
      2
      down vote










      up vote
      2
      down vote









      We have $x_n$ to be monotonically increasing and since $dfrac{x_{n+1}}{x_n}=2+dfrac{x_{n-1}}{x_n}$, we can say $dfrac{x_{n+1}}{x_n}<3$ as $dfrac{x_{n-1}}{x_n}<1$ (monotoniclly increasing)$,:forall:ngeq1$. So $x_n$ is convergent.



      Let $x_n=Acdot a^n,:aneq0$. The recurrence relation becomes $$ Acdot a^{n+1}=2Acdot a^n+Acdot a^{n-1}implies Acdot a^{n-1}(a^2-2a-1)=0underset{substack{Aneq0\aneq0}}{implies} a^2-2a-1=0$$



      By quadratic formula, $a=dfrac{2pmsqrt{4+4}}{2}=1pmsqrt{2}$. Since $x_n$'s are non negative, we have $a=displaystyle lim_{ntoinfty}x_n=1+sqrt{2}$
      $rule{17cm}{0.5pt}$



      $x_n=A(1+sqrt{2})^n+B(1-sqrt{2})^n$ where $A,B$ are independent of $n$ by linear recurrence.



      So if $L=displaystylelim_{ntoinfty}dfrac{x_n}{x_{n+1}}=lim_{ntoinfty}dfrac{A(1+sqrt{2})^n+B(1-sqrt{2})^n}{A(1+sqrt{2})^{n+1}+B(1-sqrt{2})^{n+1}}=lim_{ntoinfty}dfrac{A+dfrac{B(1-sqrt{2})^n}{B(1+sqrt{2})^n}}{dfrac{A(1+sqrt{2})^{n+1}}{(1-sqrt{2})^n}+dfrac{B(1-sqrt{2})^{n+1}}{B(1+sqrt{2})^n}}=dfrac{A}{A(1+sqrt{2})}=sqrt{2}-1$.



      $left(text{Since}: |1-sqrt{2}|<1implies displaystylelim_{ntoinfty}(1-sqrt{2})^nto0right)$.



      To show that the limit $displaystylelim_{ntoinfty}dfrac{x_n}{x_{n+1}}$ exists we see that it is bounded since $$0<dfrac{x_n}{x_{n+1}}=dfrac{x_n}{2x_{n}+x_{n-1}}leqdfrac{x_n}{2x_{n}}=dfrac12 $$
      and $x_{n+1}geq2x_nimplies x_{n+1}geq x_n$ which gives $dfrac{x_n}{x_{n+1}}=dfrac{2x_{n-1}+x_{n-2}}{2x_{n}+x_{n-1}}leqdfrac{2x_{n-1}+x_{n-2}}{2x_{n}}$ i.e.$dfrac{x_n}{x_{n+1}}-dfrac{x_{n-1}}{x_{n}}leqdfrac{x_{n-2}}{2x_{n}}$ which shows monotonicity.Hence the limit exists.






      share|cite|improve this answer














      We have $x_n$ to be monotonically increasing and since $dfrac{x_{n+1}}{x_n}=2+dfrac{x_{n-1}}{x_n}$, we can say $dfrac{x_{n+1}}{x_n}<3$ as $dfrac{x_{n-1}}{x_n}<1$ (monotoniclly increasing)$,:forall:ngeq1$. So $x_n$ is convergent.



      Let $x_n=Acdot a^n,:aneq0$. The recurrence relation becomes $$ Acdot a^{n+1}=2Acdot a^n+Acdot a^{n-1}implies Acdot a^{n-1}(a^2-2a-1)=0underset{substack{Aneq0\aneq0}}{implies} a^2-2a-1=0$$



      By quadratic formula, $a=dfrac{2pmsqrt{4+4}}{2}=1pmsqrt{2}$. Since $x_n$'s are non negative, we have $a=displaystyle lim_{ntoinfty}x_n=1+sqrt{2}$
      $rule{17cm}{0.5pt}$



      $x_n=A(1+sqrt{2})^n+B(1-sqrt{2})^n$ where $A,B$ are independent of $n$ by linear recurrence.



      So if $L=displaystylelim_{ntoinfty}dfrac{x_n}{x_{n+1}}=lim_{ntoinfty}dfrac{A(1+sqrt{2})^n+B(1-sqrt{2})^n}{A(1+sqrt{2})^{n+1}+B(1-sqrt{2})^{n+1}}=lim_{ntoinfty}dfrac{A+dfrac{B(1-sqrt{2})^n}{B(1+sqrt{2})^n}}{dfrac{A(1+sqrt{2})^{n+1}}{(1-sqrt{2})^n}+dfrac{B(1-sqrt{2})^{n+1}}{B(1+sqrt{2})^n}}=dfrac{A}{A(1+sqrt{2})}=sqrt{2}-1$.



      $left(text{Since}: |1-sqrt{2}|<1implies displaystylelim_{ntoinfty}(1-sqrt{2})^nto0right)$.



      To show that the limit $displaystylelim_{ntoinfty}dfrac{x_n}{x_{n+1}}$ exists we see that it is bounded since $$0<dfrac{x_n}{x_{n+1}}=dfrac{x_n}{2x_{n}+x_{n-1}}leqdfrac{x_n}{2x_{n}}=dfrac12 $$
      and $x_{n+1}geq2x_nimplies x_{n+1}geq x_n$ which gives $dfrac{x_n}{x_{n+1}}=dfrac{2x_{n-1}+x_{n-2}}{2x_{n}+x_{n-1}}leqdfrac{2x_{n-1}+x_{n-2}}{2x_{n}}$ i.e.$dfrac{x_n}{x_{n+1}}-dfrac{x_{n-1}}{x_{n}}leqdfrac{x_{n-2}}{2x_{n}}$ which shows monotonicity.Hence the limit exists.







      share|cite|improve this answer














      share|cite|improve this answer



      share|cite|improve this answer








      edited Nov 27 at 5:11

























      answered Nov 26 at 12:43









      Yadati Kiran

      1,243417




      1,243417












      • Wrong. The sequence of ratios is not monotonic. $x_1=1$, $x_2=2$, $x_3=5$, $x_4=12$, $x_5=29$ et cetera. Check for yourself that $x_3/x_2$ is larger than $x_4/x_3$ but also $x_5/x_4$ is larger than $x_4/x_3$. Those ratios jump around the limit approaching it from both sides.
        – Jyrki Lahtonen
        Nov 27 at 4:05










      • @JyrkiLahtonen: I was talking of the sequence ${x_n}$ not the sequence of ratios. I have tried to show the limit of the sequence ${x_n}$ and then tried to estimate the limit of the sequence of ratios.
        – Yadati Kiran
        Nov 27 at 4:19












      • Ok. But $(x_n)$ being monotonic does not imply that $(x_{n-1}/x_n)$ converges. Neither does $(x_n)$, so exactly what are you trying to say in the first paragraph?
        – Jyrki Lahtonen
        Nov 27 at 4:21












      • I never said that. I have assumed that the limit exists but yes I agree I have to show it does. The answer is incomplete.
        – Yadati Kiran
        Nov 27 at 4:25












      • Ok. I do approve of your solution of actually solving the recurrence. That does provide the rigor (if you show that $Aneq0$). Adding my upvote. IMHO your answer would be better without that first paragraph :-)
        – Jyrki Lahtonen
        Nov 27 at 4:30




















      • Wrong. The sequence of ratios is not monotonic. $x_1=1$, $x_2=2$, $x_3=5$, $x_4=12$, $x_5=29$ et cetera. Check for yourself that $x_3/x_2$ is larger than $x_4/x_3$ but also $x_5/x_4$ is larger than $x_4/x_3$. Those ratios jump around the limit approaching it from both sides.
        – Jyrki Lahtonen
        Nov 27 at 4:05










      • @JyrkiLahtonen: I was talking of the sequence ${x_n}$ not the sequence of ratios. I have tried to show the limit of the sequence ${x_n}$ and then tried to estimate the limit of the sequence of ratios.
        – Yadati Kiran
        Nov 27 at 4:19












      • Ok. But $(x_n)$ being monotonic does not imply that $(x_{n-1}/x_n)$ converges. Neither does $(x_n)$, so exactly what are you trying to say in the first paragraph?
        – Jyrki Lahtonen
        Nov 27 at 4:21












      • I never said that. I have assumed that the limit exists but yes I agree I have to show it does. The answer is incomplete.
        – Yadati Kiran
        Nov 27 at 4:25












      • Ok. I do approve of your solution of actually solving the recurrence. That does provide the rigor (if you show that $Aneq0$). Adding my upvote. IMHO your answer would be better without that first paragraph :-)
        – Jyrki Lahtonen
        Nov 27 at 4:30


















      Wrong. The sequence of ratios is not monotonic. $x_1=1$, $x_2=2$, $x_3=5$, $x_4=12$, $x_5=29$ et cetera. Check for yourself that $x_3/x_2$ is larger than $x_4/x_3$ but also $x_5/x_4$ is larger than $x_4/x_3$. Those ratios jump around the limit approaching it from both sides.
      – Jyrki Lahtonen
      Nov 27 at 4:05




      Wrong. The sequence of ratios is not monotonic. $x_1=1$, $x_2=2$, $x_3=5$, $x_4=12$, $x_5=29$ et cetera. Check for yourself that $x_3/x_2$ is larger than $x_4/x_3$ but also $x_5/x_4$ is larger than $x_4/x_3$. Those ratios jump around the limit approaching it from both sides.
      – Jyrki Lahtonen
      Nov 27 at 4:05












      @JyrkiLahtonen: I was talking of the sequence ${x_n}$ not the sequence of ratios. I have tried to show the limit of the sequence ${x_n}$ and then tried to estimate the limit of the sequence of ratios.
      – Yadati Kiran
      Nov 27 at 4:19






      @JyrkiLahtonen: I was talking of the sequence ${x_n}$ not the sequence of ratios. I have tried to show the limit of the sequence ${x_n}$ and then tried to estimate the limit of the sequence of ratios.
      – Yadati Kiran
      Nov 27 at 4:19














      Ok. But $(x_n)$ being monotonic does not imply that $(x_{n-1}/x_n)$ converges. Neither does $(x_n)$, so exactly what are you trying to say in the first paragraph?
      – Jyrki Lahtonen
      Nov 27 at 4:21






      Ok. But $(x_n)$ being monotonic does not imply that $(x_{n-1}/x_n)$ converges. Neither does $(x_n)$, so exactly what are you trying to say in the first paragraph?
      – Jyrki Lahtonen
      Nov 27 at 4:21














      I never said that. I have assumed that the limit exists but yes I agree I have to show it does. The answer is incomplete.
      – Yadati Kiran
      Nov 27 at 4:25






      I never said that. I have assumed that the limit exists but yes I agree I have to show it does. The answer is incomplete.
      – Yadati Kiran
      Nov 27 at 4:25














      Ok. I do approve of your solution of actually solving the recurrence. That does provide the rigor (if you show that $Aneq0$). Adding my upvote. IMHO your answer would be better without that first paragraph :-)
      – Jyrki Lahtonen
      Nov 27 at 4:30






      Ok. I do approve of your solution of actually solving the recurrence. That does provide the rigor (if you show that $Aneq0$). Adding my upvote. IMHO your answer would be better without that first paragraph :-)
      – Jyrki Lahtonen
      Nov 27 at 4:30












      up vote
      1
      down vote













      One trick is to use the given limit to derive the existence of it.



      Write $y_n=frac{x_n}{x_{n+1}}$.




      Claim: There exists $rin (0,1)$ such that
      $$(sqrt 2-1)-y_nle rbig(y_{n-1}-(sqrt2-1)big).$$




      Proof: Rearranging the terms,
      begin{align*}
      underbrace{frac{x_{n+1}}{x_n}}_{=1/ {y_n}}-1&=underbrace{frac{x_{n-1}}{x_n}}_{=y_{n-1}}+1\
      frac 1{y_n}-(sqrt2+1)&=y_{n-1}-(sqrt2-1)\
      frac 1{y_n}-frac1{sqrt2-1}&=y_{n-1}-(sqrt2-1)\
      (sqrt 2-1)-y_n&=underbrace{y_n(sqrt2-1)}_{in(0,1)}big(y_{n-1}-(sqrt2-1)big).
      end{align*}

      so the claim is true. $square$



      Therefore,
      begin{align*}
      |y_n-(sqrt2-1)|&le r |y_{n-1}-(sqrt2-1)|le r^2 |y_{n-2}-(sqrt2-1)|lecdots\
      &le r^{n}|y_0-(sqrt2-1)|to0,
      end{align*}

      implies the limit exists, and $limlimits_{ntoinfty}y_n=sqrt2-1$.






      share|cite|improve this answer



























        up vote
        1
        down vote













        One trick is to use the given limit to derive the existence of it.



        Write $y_n=frac{x_n}{x_{n+1}}$.




        Claim: There exists $rin (0,1)$ such that
        $$(sqrt 2-1)-y_nle rbig(y_{n-1}-(sqrt2-1)big).$$




        Proof: Rearranging the terms,
        begin{align*}
        underbrace{frac{x_{n+1}}{x_n}}_{=1/ {y_n}}-1&=underbrace{frac{x_{n-1}}{x_n}}_{=y_{n-1}}+1\
        frac 1{y_n}-(sqrt2+1)&=y_{n-1}-(sqrt2-1)\
        frac 1{y_n}-frac1{sqrt2-1}&=y_{n-1}-(sqrt2-1)\
        (sqrt 2-1)-y_n&=underbrace{y_n(sqrt2-1)}_{in(0,1)}big(y_{n-1}-(sqrt2-1)big).
        end{align*}

        so the claim is true. $square$



        Therefore,
        begin{align*}
        |y_n-(sqrt2-1)|&le r |y_{n-1}-(sqrt2-1)|le r^2 |y_{n-2}-(sqrt2-1)|lecdots\
        &le r^{n}|y_0-(sqrt2-1)|to0,
        end{align*}

        implies the limit exists, and $limlimits_{ntoinfty}y_n=sqrt2-1$.






        share|cite|improve this answer

























          up vote
          1
          down vote










          up vote
          1
          down vote









          One trick is to use the given limit to derive the existence of it.



          Write $y_n=frac{x_n}{x_{n+1}}$.




          Claim: There exists $rin (0,1)$ such that
          $$(sqrt 2-1)-y_nle rbig(y_{n-1}-(sqrt2-1)big).$$




          Proof: Rearranging the terms,
          begin{align*}
          underbrace{frac{x_{n+1}}{x_n}}_{=1/ {y_n}}-1&=underbrace{frac{x_{n-1}}{x_n}}_{=y_{n-1}}+1\
          frac 1{y_n}-(sqrt2+1)&=y_{n-1}-(sqrt2-1)\
          frac 1{y_n}-frac1{sqrt2-1}&=y_{n-1}-(sqrt2-1)\
          (sqrt 2-1)-y_n&=underbrace{y_n(sqrt2-1)}_{in(0,1)}big(y_{n-1}-(sqrt2-1)big).
          end{align*}

          so the claim is true. $square$



          Therefore,
          begin{align*}
          |y_n-(sqrt2-1)|&le r |y_{n-1}-(sqrt2-1)|le r^2 |y_{n-2}-(sqrt2-1)|lecdots\
          &le r^{n}|y_0-(sqrt2-1)|to0,
          end{align*}

          implies the limit exists, and $limlimits_{ntoinfty}y_n=sqrt2-1$.






          share|cite|improve this answer














          One trick is to use the given limit to derive the existence of it.



          Write $y_n=frac{x_n}{x_{n+1}}$.




          Claim: There exists $rin (0,1)$ such that
          $$(sqrt 2-1)-y_nle rbig(y_{n-1}-(sqrt2-1)big).$$




          Proof: Rearranging the terms,
          begin{align*}
          underbrace{frac{x_{n+1}}{x_n}}_{=1/ {y_n}}-1&=underbrace{frac{x_{n-1}}{x_n}}_{=y_{n-1}}+1\
          frac 1{y_n}-(sqrt2+1)&=y_{n-1}-(sqrt2-1)\
          frac 1{y_n}-frac1{sqrt2-1}&=y_{n-1}-(sqrt2-1)\
          (sqrt 2-1)-y_n&=underbrace{y_n(sqrt2-1)}_{in(0,1)}big(y_{n-1}-(sqrt2-1)big).
          end{align*}

          so the claim is true. $square$



          Therefore,
          begin{align*}
          |y_n-(sqrt2-1)|&le r |y_{n-1}-(sqrt2-1)|le r^2 |y_{n-2}-(sqrt2-1)|lecdots\
          &le r^{n}|y_0-(sqrt2-1)|to0,
          end{align*}

          implies the limit exists, and $limlimits_{ntoinfty}y_n=sqrt2-1$.







          share|cite|improve this answer














          share|cite|improve this answer



          share|cite|improve this answer








          edited Nov 26 at 12:55

























          answered Nov 26 at 12:30









          Tianlalu

          2,9011935




          2,9011935






















              up vote
              0
              down vote













              Look at $x_n/x_{n+1} - x_{n-1}/x_n$ which is $(x_n^2 - x_{n-1}x_{n+1})/x_n x_{n-1}$.
              We can prove by induction that the numerator is $(-1)^n$.
              $$x_{n+1}^2 - x_n x_{n+2}
              = x_{n+1}^2 - x_n(2x_{n+1} + x_n)
              = x_{n+1}(x_{n+1} - 2x_n) - x_n^2
              = -(x_n^2 - x_{n-1}x_{n+1})$$

              with $x_1^2 - x_0x_2 = -1$. Hence $x_n/x_{n+1}$ tends to a limit by the alternating series test.






              share|cite|improve this answer





















              • You may want to use frac{}{} to write fractions more clearly.
                – DonAntonio
                Nov 26 at 12:58















              up vote
              0
              down vote













              Look at $x_n/x_{n+1} - x_{n-1}/x_n$ which is $(x_n^2 - x_{n-1}x_{n+1})/x_n x_{n-1}$.
              We can prove by induction that the numerator is $(-1)^n$.
              $$x_{n+1}^2 - x_n x_{n+2}
              = x_{n+1}^2 - x_n(2x_{n+1} + x_n)
              = x_{n+1}(x_{n+1} - 2x_n) - x_n^2
              = -(x_n^2 - x_{n-1}x_{n+1})$$

              with $x_1^2 - x_0x_2 = -1$. Hence $x_n/x_{n+1}$ tends to a limit by the alternating series test.






              share|cite|improve this answer





















              • You may want to use frac{}{} to write fractions more clearly.
                – DonAntonio
                Nov 26 at 12:58













              up vote
              0
              down vote










              up vote
              0
              down vote









              Look at $x_n/x_{n+1} - x_{n-1}/x_n$ which is $(x_n^2 - x_{n-1}x_{n+1})/x_n x_{n-1}$.
              We can prove by induction that the numerator is $(-1)^n$.
              $$x_{n+1}^2 - x_n x_{n+2}
              = x_{n+1}^2 - x_n(2x_{n+1} + x_n)
              = x_{n+1}(x_{n+1} - 2x_n) - x_n^2
              = -(x_n^2 - x_{n-1}x_{n+1})$$

              with $x_1^2 - x_0x_2 = -1$. Hence $x_n/x_{n+1}$ tends to a limit by the alternating series test.






              share|cite|improve this answer












              Look at $x_n/x_{n+1} - x_{n-1}/x_n$ which is $(x_n^2 - x_{n-1}x_{n+1})/x_n x_{n-1}$.
              We can prove by induction that the numerator is $(-1)^n$.
              $$x_{n+1}^2 - x_n x_{n+2}
              = x_{n+1}^2 - x_n(2x_{n+1} + x_n)
              = x_{n+1}(x_{n+1} - 2x_n) - x_n^2
              = -(x_n^2 - x_{n-1}x_{n+1})$$

              with $x_1^2 - x_0x_2 = -1$. Hence $x_n/x_{n+1}$ tends to a limit by the alternating series test.







              share|cite|improve this answer












              share|cite|improve this answer



              share|cite|improve this answer










              answered Nov 26 at 12:57









              Michael Behrend

              1,04746




              1,04746












              • You may want to use frac{}{} to write fractions more clearly.
                – DonAntonio
                Nov 26 at 12:58


















              • You may want to use frac{}{} to write fractions more clearly.
                – DonAntonio
                Nov 26 at 12:58
















              You may want to use frac{}{} to write fractions more clearly.
              – DonAntonio
              Nov 26 at 12:58




              You may want to use frac{}{} to write fractions more clearly.
              – DonAntonio
              Nov 26 at 12:58










              up vote
              0
              down vote













              Since $x_n$ is increasing, all the terms become non-zero. By defining $a_n={x_nover x_{n+1}}$ we have $${x_nover x_{n+1}}={x_nover 2x_n+x_{n-1}}={1over 2+{x_{n-1}over {x_n}}}$$therefore $$a_n={1over 2+{ a_{n-1}}}$$Now by defining $b_n=a_n-(sqrt 2-1)$ we have $$b_n+sqrt 2-1={1over b_{n-1}+sqrt 2+1}$$therefore $$b_n=-sqrt 2+1+{1over b_{n-1}+sqrt 2+1}={(1-sqrt 2)b_{n-1}over a_n+2}$$since $x_n>0$ we have $a_n>0$ therefore$$|b_n|=|{(1-sqrt 2)b_{n-1}over a_n+2}|le {sqrt 2-1over 2}|b_{n-1}|$$which means that $b_n to 0$ or $a_n={x_nover x_{n+1}}to sqrt 2-1 quadblacksquare$






              share|cite|improve this answer

























                up vote
                0
                down vote













                Since $x_n$ is increasing, all the terms become non-zero. By defining $a_n={x_nover x_{n+1}}$ we have $${x_nover x_{n+1}}={x_nover 2x_n+x_{n-1}}={1over 2+{x_{n-1}over {x_n}}}$$therefore $$a_n={1over 2+{ a_{n-1}}}$$Now by defining $b_n=a_n-(sqrt 2-1)$ we have $$b_n+sqrt 2-1={1over b_{n-1}+sqrt 2+1}$$therefore $$b_n=-sqrt 2+1+{1over b_{n-1}+sqrt 2+1}={(1-sqrt 2)b_{n-1}over a_n+2}$$since $x_n>0$ we have $a_n>0$ therefore$$|b_n|=|{(1-sqrt 2)b_{n-1}over a_n+2}|le {sqrt 2-1over 2}|b_{n-1}|$$which means that $b_n to 0$ or $a_n={x_nover x_{n+1}}to sqrt 2-1 quadblacksquare$






                share|cite|improve this answer























                  up vote
                  0
                  down vote










                  up vote
                  0
                  down vote









                  Since $x_n$ is increasing, all the terms become non-zero. By defining $a_n={x_nover x_{n+1}}$ we have $${x_nover x_{n+1}}={x_nover 2x_n+x_{n-1}}={1over 2+{x_{n-1}over {x_n}}}$$therefore $$a_n={1over 2+{ a_{n-1}}}$$Now by defining $b_n=a_n-(sqrt 2-1)$ we have $$b_n+sqrt 2-1={1over b_{n-1}+sqrt 2+1}$$therefore $$b_n=-sqrt 2+1+{1over b_{n-1}+sqrt 2+1}={(1-sqrt 2)b_{n-1}over a_n+2}$$since $x_n>0$ we have $a_n>0$ therefore$$|b_n|=|{(1-sqrt 2)b_{n-1}over a_n+2}|le {sqrt 2-1over 2}|b_{n-1}|$$which means that $b_n to 0$ or $a_n={x_nover x_{n+1}}to sqrt 2-1 quadblacksquare$






                  share|cite|improve this answer












                  Since $x_n$ is increasing, all the terms become non-zero. By defining $a_n={x_nover x_{n+1}}$ we have $${x_nover x_{n+1}}={x_nover 2x_n+x_{n-1}}={1over 2+{x_{n-1}over {x_n}}}$$therefore $$a_n={1over 2+{ a_{n-1}}}$$Now by defining $b_n=a_n-(sqrt 2-1)$ we have $$b_n+sqrt 2-1={1over b_{n-1}+sqrt 2+1}$$therefore $$b_n=-sqrt 2+1+{1over b_{n-1}+sqrt 2+1}={(1-sqrt 2)b_{n-1}over a_n+2}$$since $x_n>0$ we have $a_n>0$ therefore$$|b_n|=|{(1-sqrt 2)b_{n-1}over a_n+2}|le {sqrt 2-1over 2}|b_{n-1}|$$which means that $b_n to 0$ or $a_n={x_nover x_{n+1}}to sqrt 2-1 quadblacksquare$







                  share|cite|improve this answer












                  share|cite|improve this answer



                  share|cite|improve this answer










                  answered Nov 28 at 0:25









                  Mostafa Ayaz

                  13.3k3836




                  13.3k3836






















                      up vote
                      -1
                      down vote













                      We have that



                      $$frac{x_{n+1}}{x_n} = 2 + frac{x_{n-1}}{x_n}$$



                      then by



                      $$y_n=frac{x_{n}}{x_{n-1}} implies y_{n+1}=2+frac1{y_n }quad y_0=2$$



                      which converges to $L=sqrt 2+1$ and then



                      $$lim_{nto infty} frac{x_n}{x_{n+1}}=lim_{nto infty} frac{1}{y_{n+1}}=frac1L=sqrt 2 -1$$






                      share|cite|improve this answer



















                      • 1




                        It seems $y_n$ is 'alternating', i.e. increasing/decreasing when $n$ even and decreasing/increasing when $n$ odd.
                        – Tianlalu
                        Nov 26 at 13:08






                      • 2




                        Why $;y_n;$ is increasing and bounded?
                        – DonAntonio
                        Nov 26 at 13:10










                      • @DonAntonio Yes I need to clarify that point better! Thanks
                        – gimusi
                        Nov 26 at 13:17















                      up vote
                      -1
                      down vote













                      We have that



                      $$frac{x_{n+1}}{x_n} = 2 + frac{x_{n-1}}{x_n}$$



                      then by



                      $$y_n=frac{x_{n}}{x_{n-1}} implies y_{n+1}=2+frac1{y_n }quad y_0=2$$



                      which converges to $L=sqrt 2+1$ and then



                      $$lim_{nto infty} frac{x_n}{x_{n+1}}=lim_{nto infty} frac{1}{y_{n+1}}=frac1L=sqrt 2 -1$$






                      share|cite|improve this answer



















                      • 1




                        It seems $y_n$ is 'alternating', i.e. increasing/decreasing when $n$ even and decreasing/increasing when $n$ odd.
                        – Tianlalu
                        Nov 26 at 13:08






                      • 2




                        Why $;y_n;$ is increasing and bounded?
                        – DonAntonio
                        Nov 26 at 13:10










                      • @DonAntonio Yes I need to clarify that point better! Thanks
                        – gimusi
                        Nov 26 at 13:17













                      up vote
                      -1
                      down vote










                      up vote
                      -1
                      down vote









                      We have that



                      $$frac{x_{n+1}}{x_n} = 2 + frac{x_{n-1}}{x_n}$$



                      then by



                      $$y_n=frac{x_{n}}{x_{n-1}} implies y_{n+1}=2+frac1{y_n }quad y_0=2$$



                      which converges to $L=sqrt 2+1$ and then



                      $$lim_{nto infty} frac{x_n}{x_{n+1}}=lim_{nto infty} frac{1}{y_{n+1}}=frac1L=sqrt 2 -1$$






                      share|cite|improve this answer














                      We have that



                      $$frac{x_{n+1}}{x_n} = 2 + frac{x_{n-1}}{x_n}$$



                      then by



                      $$y_n=frac{x_{n}}{x_{n-1}} implies y_{n+1}=2+frac1{y_n }quad y_0=2$$



                      which converges to $L=sqrt 2+1$ and then



                      $$lim_{nto infty} frac{x_n}{x_{n+1}}=lim_{nto infty} frac{1}{y_{n+1}}=frac1L=sqrt 2 -1$$







                      share|cite|improve this answer














                      share|cite|improve this answer



                      share|cite|improve this answer








                      edited Nov 26 at 13:21

























                      answered Nov 26 at 13:04









                      gimusi

                      89.7k74495




                      89.7k74495








                      • 1




                        It seems $y_n$ is 'alternating', i.e. increasing/decreasing when $n$ even and decreasing/increasing when $n$ odd.
                        – Tianlalu
                        Nov 26 at 13:08






                      • 2




                        Why $;y_n;$ is increasing and bounded?
                        – DonAntonio
                        Nov 26 at 13:10










                      • @DonAntonio Yes I need to clarify that point better! Thanks
                        – gimusi
                        Nov 26 at 13:17














                      • 1




                        It seems $y_n$ is 'alternating', i.e. increasing/decreasing when $n$ even and decreasing/increasing when $n$ odd.
                        – Tianlalu
                        Nov 26 at 13:08






                      • 2




                        Why $;y_n;$ is increasing and bounded?
                        – DonAntonio
                        Nov 26 at 13:10










                      • @DonAntonio Yes I need to clarify that point better! Thanks
                        – gimusi
                        Nov 26 at 13:17








                      1




                      1




                      It seems $y_n$ is 'alternating', i.e. increasing/decreasing when $n$ even and decreasing/increasing when $n$ odd.
                      – Tianlalu
                      Nov 26 at 13:08




                      It seems $y_n$ is 'alternating', i.e. increasing/decreasing when $n$ even and decreasing/increasing when $n$ odd.
                      – Tianlalu
                      Nov 26 at 13:08




                      2




                      2




                      Why $;y_n;$ is increasing and bounded?
                      – DonAntonio
                      Nov 26 at 13:10




                      Why $;y_n;$ is increasing and bounded?
                      – DonAntonio
                      Nov 26 at 13:10












                      @DonAntonio Yes I need to clarify that point better! Thanks
                      – gimusi
                      Nov 26 at 13:17




                      @DonAntonio Yes I need to clarify that point better! Thanks
                      – gimusi
                      Nov 26 at 13:17










                      up vote
                      -1
                      down vote













                      Let



                      $$lim_{nto infty} frac{x_n}{x_{n+1}} = lim_{nto infty} frac{x_{n-1}}{x_{n1}} = k$$



                      Now



                      $$lim_{nto infty} frac{x_n}{x_{n+1}} = k$$
                      $$lim_{nto infty} frac{x_n}{2x_{n} + x_{n-1}} = k$$



                      Take $x_n$ out from numerator and denominator
                      $$lim_{nto infty} frac{1}{2 + frac{x_{n-1}}{x_n}} = k$$



                      Using the first equation



                      $$ frac{1}{2 + k} = k$$
                      $$k^2+2k-1=0$$
                      This gives two solutions $k=sqrt{2}-1$ and $k=-sqrt{2}-1$. Since none of the terms can be negative, we reject . the second solution thereby giving us



                      $$lim_{nto infty} frac{x_n}{x_{n+1}} = k = sqrt2 - 1$$



                      EDIT - As suggested by the commenter we need to prove that it is a finite limit before we start with the proof. Initially it's a $frac{infty}{infty}$ form as both $x_n$ and $x_{n+1}$ approach $infty$ as $n$ approaches $infty$. I'll use a finite upper bound to show that the limit is finite which means it exists.



                      For any $n$



                      $$frac{x_n}{x_{n+1}} =frac{x_n}{2x_n + x_{n-1}}$$



                      As $x_{n-1}$ is always a positive quantity
                      $$frac{x_n}{x_{n+1}} leq frac{x_n}{2x_n}$$
                      $$frac{x_n}{x_{n+1}} leq frac{1}{2}$$



                      For any $n$, you can take the last statement to prove the monotonicity as
                      $$x_{n+1}geq x_n$$
                      And since both $x_n$ and $x_{n+1}$ are positive values, the lower bound is $0$. The upper bound along with lower bound and the monotonicity proves that the limit is finite.






                      share|cite|improve this answer



















                      • 3




                        This seems to be the less hard part of the question. The hardest part is to prove the limit exists...
                        – DonAntonio
                        Nov 26 at 12:22






                      • 1




                        Is there any way to independently do that ?
                        – Sauhard Sharma
                        Nov 26 at 12:35










                      • Why can you write the first line? why is the limit of $frac{x_n}{x_{n+1}} = frac{x_{n-1}}{x_n}$ ?
                        – M-S-R
                        Nov 26 at 12:54












                      • @SauhardSharma It better is, otherwise your whole answer is invalid as you can use arithmetic of limits only if you know that the limit exists finitely, otherwise you can't. You can try induction, for example, to prove monotonicity or something like that
                        – DonAntonio
                        Nov 26 at 13:02












                      • @DonAntonio Thanks for pointing out my mistake. I think the edited proof should suffice
                        – Sauhard Sharma
                        Nov 26 at 13:18















                      up vote
                      -1
                      down vote













                      Let



                      $$lim_{nto infty} frac{x_n}{x_{n+1}} = lim_{nto infty} frac{x_{n-1}}{x_{n1}} = k$$



                      Now



                      $$lim_{nto infty} frac{x_n}{x_{n+1}} = k$$
                      $$lim_{nto infty} frac{x_n}{2x_{n} + x_{n-1}} = k$$



                      Take $x_n$ out from numerator and denominator
                      $$lim_{nto infty} frac{1}{2 + frac{x_{n-1}}{x_n}} = k$$



                      Using the first equation



                      $$ frac{1}{2 + k} = k$$
                      $$k^2+2k-1=0$$
                      This gives two solutions $k=sqrt{2}-1$ and $k=-sqrt{2}-1$. Since none of the terms can be negative, we reject . the second solution thereby giving us



                      $$lim_{nto infty} frac{x_n}{x_{n+1}} = k = sqrt2 - 1$$



                      EDIT - As suggested by the commenter we need to prove that it is a finite limit before we start with the proof. Initially it's a $frac{infty}{infty}$ form as both $x_n$ and $x_{n+1}$ approach $infty$ as $n$ approaches $infty$. I'll use a finite upper bound to show that the limit is finite which means it exists.



                      For any $n$



                      $$frac{x_n}{x_{n+1}} =frac{x_n}{2x_n + x_{n-1}}$$



                      As $x_{n-1}$ is always a positive quantity
                      $$frac{x_n}{x_{n+1}} leq frac{x_n}{2x_n}$$
                      $$frac{x_n}{x_{n+1}} leq frac{1}{2}$$



                      For any $n$, you can take the last statement to prove the monotonicity as
                      $$x_{n+1}geq x_n$$
                      And since both $x_n$ and $x_{n+1}$ are positive values, the lower bound is $0$. The upper bound along with lower bound and the monotonicity proves that the limit is finite.






                      share|cite|improve this answer



















                      • 3




                        This seems to be the less hard part of the question. The hardest part is to prove the limit exists...
                        – DonAntonio
                        Nov 26 at 12:22






                      • 1




                        Is there any way to independently do that ?
                        – Sauhard Sharma
                        Nov 26 at 12:35










                      • Why can you write the first line? why is the limit of $frac{x_n}{x_{n+1}} = frac{x_{n-1}}{x_n}$ ?
                        – M-S-R
                        Nov 26 at 12:54












                      • @SauhardSharma It better is, otherwise your whole answer is invalid as you can use arithmetic of limits only if you know that the limit exists finitely, otherwise you can't. You can try induction, for example, to prove monotonicity or something like that
                        – DonAntonio
                        Nov 26 at 13:02












                      • @DonAntonio Thanks for pointing out my mistake. I think the edited proof should suffice
                        – Sauhard Sharma
                        Nov 26 at 13:18













                      up vote
                      -1
                      down vote










                      up vote
                      -1
                      down vote









                      Let



                      $$lim_{nto infty} frac{x_n}{x_{n+1}} = lim_{nto infty} frac{x_{n-1}}{x_{n1}} = k$$



                      Now



                      $$lim_{nto infty} frac{x_n}{x_{n+1}} = k$$
                      $$lim_{nto infty} frac{x_n}{2x_{n} + x_{n-1}} = k$$



                      Take $x_n$ out from numerator and denominator
                      $$lim_{nto infty} frac{1}{2 + frac{x_{n-1}}{x_n}} = k$$



                      Using the first equation



                      $$ frac{1}{2 + k} = k$$
                      $$k^2+2k-1=0$$
                      This gives two solutions $k=sqrt{2}-1$ and $k=-sqrt{2}-1$. Since none of the terms can be negative, we reject . the second solution thereby giving us



                      $$lim_{nto infty} frac{x_n}{x_{n+1}} = k = sqrt2 - 1$$



                      EDIT - As suggested by the commenter we need to prove that it is a finite limit before we start with the proof. Initially it's a $frac{infty}{infty}$ form as both $x_n$ and $x_{n+1}$ approach $infty$ as $n$ approaches $infty$. I'll use a finite upper bound to show that the limit is finite which means it exists.



                      For any $n$



                      $$frac{x_n}{x_{n+1}} =frac{x_n}{2x_n + x_{n-1}}$$



                      As $x_{n-1}$ is always a positive quantity
                      $$frac{x_n}{x_{n+1}} leq frac{x_n}{2x_n}$$
                      $$frac{x_n}{x_{n+1}} leq frac{1}{2}$$



                      For any $n$, you can take the last statement to prove the monotonicity as
                      $$x_{n+1}geq x_n$$
                      And since both $x_n$ and $x_{n+1}$ are positive values, the lower bound is $0$. The upper bound along with lower bound and the monotonicity proves that the limit is finite.






                      share|cite|improve this answer














                      Let



                      $$lim_{nto infty} frac{x_n}{x_{n+1}} = lim_{nto infty} frac{x_{n-1}}{x_{n1}} = k$$



                      Now



                      $$lim_{nto infty} frac{x_n}{x_{n+1}} = k$$
                      $$lim_{nto infty} frac{x_n}{2x_{n} + x_{n-1}} = k$$



                      Take $x_n$ out from numerator and denominator
                      $$lim_{nto infty} frac{1}{2 + frac{x_{n-1}}{x_n}} = k$$



                      Using the first equation



                      $$ frac{1}{2 + k} = k$$
                      $$k^2+2k-1=0$$
                      This gives two solutions $k=sqrt{2}-1$ and $k=-sqrt{2}-1$. Since none of the terms can be negative, we reject . the second solution thereby giving us



                      $$lim_{nto infty} frac{x_n}{x_{n+1}} = k = sqrt2 - 1$$



                      EDIT - As suggested by the commenter we need to prove that it is a finite limit before we start with the proof. Initially it's a $frac{infty}{infty}$ form as both $x_n$ and $x_{n+1}$ approach $infty$ as $n$ approaches $infty$. I'll use a finite upper bound to show that the limit is finite which means it exists.



                      For any $n$



                      $$frac{x_n}{x_{n+1}} =frac{x_n}{2x_n + x_{n-1}}$$



                      As $x_{n-1}$ is always a positive quantity
                      $$frac{x_n}{x_{n+1}} leq frac{x_n}{2x_n}$$
                      $$frac{x_n}{x_{n+1}} leq frac{1}{2}$$



                      For any $n$, you can take the last statement to prove the monotonicity as
                      $$x_{n+1}geq x_n$$
                      And since both $x_n$ and $x_{n+1}$ are positive values, the lower bound is $0$. The upper bound along with lower bound and the monotonicity proves that the limit is finite.







                      share|cite|improve this answer














                      share|cite|improve this answer



                      share|cite|improve this answer








                      edited Nov 26 at 13:48

























                      answered Nov 26 at 12:18









                      Sauhard Sharma

                      3538




                      3538








                      • 3




                        This seems to be the less hard part of the question. The hardest part is to prove the limit exists...
                        – DonAntonio
                        Nov 26 at 12:22






                      • 1




                        Is there any way to independently do that ?
                        – Sauhard Sharma
                        Nov 26 at 12:35










                      • Why can you write the first line? why is the limit of $frac{x_n}{x_{n+1}} = frac{x_{n-1}}{x_n}$ ?
                        – M-S-R
                        Nov 26 at 12:54












                      • @SauhardSharma It better is, otherwise your whole answer is invalid as you can use arithmetic of limits only if you know that the limit exists finitely, otherwise you can't. You can try induction, for example, to prove monotonicity or something like that
                        – DonAntonio
                        Nov 26 at 13:02












                      • @DonAntonio Thanks for pointing out my mistake. I think the edited proof should suffice
                        – Sauhard Sharma
                        Nov 26 at 13:18














                      • 3




                        This seems to be the less hard part of the question. The hardest part is to prove the limit exists...
                        – DonAntonio
                        Nov 26 at 12:22






                      • 1




                        Is there any way to independently do that ?
                        – Sauhard Sharma
                        Nov 26 at 12:35










                      • Why can you write the first line? why is the limit of $frac{x_n}{x_{n+1}} = frac{x_{n-1}}{x_n}$ ?
                        – M-S-R
                        Nov 26 at 12:54












                      • @SauhardSharma It better is, otherwise your whole answer is invalid as you can use arithmetic of limits only if you know that the limit exists finitely, otherwise you can't. You can try induction, for example, to prove monotonicity or something like that
                        – DonAntonio
                        Nov 26 at 13:02












                      • @DonAntonio Thanks for pointing out my mistake. I think the edited proof should suffice
                        – Sauhard Sharma
                        Nov 26 at 13:18








                      3




                      3




                      This seems to be the less hard part of the question. The hardest part is to prove the limit exists...
                      – DonAntonio
                      Nov 26 at 12:22




                      This seems to be the less hard part of the question. The hardest part is to prove the limit exists...
                      – DonAntonio
                      Nov 26 at 12:22




                      1




                      1




                      Is there any way to independently do that ?
                      – Sauhard Sharma
                      Nov 26 at 12:35




                      Is there any way to independently do that ?
                      – Sauhard Sharma
                      Nov 26 at 12:35












                      Why can you write the first line? why is the limit of $frac{x_n}{x_{n+1}} = frac{x_{n-1}}{x_n}$ ?
                      – M-S-R
                      Nov 26 at 12:54






                      Why can you write the first line? why is the limit of $frac{x_n}{x_{n+1}} = frac{x_{n-1}}{x_n}$ ?
                      – M-S-R
                      Nov 26 at 12:54














                      @SauhardSharma It better is, otherwise your whole answer is invalid as you can use arithmetic of limits only if you know that the limit exists finitely, otherwise you can't. You can try induction, for example, to prove monotonicity or something like that
                      – DonAntonio
                      Nov 26 at 13:02






                      @SauhardSharma It better is, otherwise your whole answer is invalid as you can use arithmetic of limits only if you know that the limit exists finitely, otherwise you can't. You can try induction, for example, to prove monotonicity or something like that
                      – DonAntonio
                      Nov 26 at 13:02














                      @DonAntonio Thanks for pointing out my mistake. I think the edited proof should suffice
                      – Sauhard Sharma
                      Nov 26 at 13:18




                      @DonAntonio Thanks for pointing out my mistake. I think the edited proof should suffice
                      – Sauhard Sharma
                      Nov 26 at 13:18


















                      draft saved

                      draft discarded




















































                      Thanks for contributing an answer to Mathematics Stack Exchange!


                      • Please be sure to answer the question. Provide details and share your research!

                      But avoid



                      • Asking for help, clarification, or responding to other answers.

                      • Making statements based on opinion; back them up with references or personal experience.


                      Use MathJax to format equations. MathJax reference.


                      To learn more, see our tips on writing great answers.





                      Some of your past answers have not been well-received, and you're in danger of being blocked from answering.


                      Please pay close attention to the following guidance:


                      • Please be sure to answer the question. Provide details and share your research!

                      But avoid



                      • Asking for help, clarification, or responding to other answers.

                      • Making statements based on opinion; back them up with references or personal experience.


                      To learn more, see our tips on writing great answers.




                      draft saved


                      draft discarded














                      StackExchange.ready(
                      function () {
                      StackExchange.openid.initPostLogin('.new-post-login', 'https%3a%2f%2fmath.stackexchange.com%2fquestions%2f3014227%2fprove-existence-of-the-limit-of-a-sequence%23new-answer', 'question_page');
                      }
                      );

                      Post as a guest















                      Required, but never shown





















































                      Required, but never shown














                      Required, but never shown












                      Required, but never shown







                      Required, but never shown

































                      Required, but never shown














                      Required, but never shown












                      Required, but never shown







                      Required, but never shown







                      Popular posts from this blog

                      If I really need a card on my start hand, how many mulligans make sense? [duplicate]

                      Alcedinidae

                      Can an atomic nucleus contain both particles and antiparticles? [duplicate]